gastro lippicott

Réussis tes devoirs et examens dès maintenant avec Quizwiz!

A 65-year-old woman presents with a 5-week history of yellow skin and sclera, anorexia, and epigastric pain. Her past medical history is signifi cant for insulin-dependent diabetes mellitus. She smoked one pack of cigarettes a day for the past 20 years. Physical examination reveals jaundice and a palpable gallbladder. Laboratory studies show a serum bilirubin level of 10 mg/dL, mostly in the conjugated form, and an elevated alkaline phosphatase (260 U/L). A CT scan of the abdomen discloses a mass in the head of the pancreas and multiple nodules in the liver measuring up to 3 cm. Which of the following is the most likely cause of jaundice in this patient? (A) Cholelithiasis (B) Cirrhosis (C) Extrahepatic biliary obstruction (D) Hemolysis (E) Intrahepatic cholestasis

The answer C: Extrahepatic biliary obstruction. Pancreatic adenocarcinomas often cause obstruction of the common bile duct due to the proximity of the duct to the head of the pancreas. Painless jaundice is a frequent initial symptom of pancreatic cancer. Dilation of the gallbladder in this setting is termed Courvoisier sign. Cirrhosis (choice B) is a late complication of extrahepatic biliary obstruction. Diagnosis: Pancreatic adenocarcinoma

A 45-year-old, mildly obese woman presents with a 1-week history of upper abdominal pain, fever, shaking chills, and occasional vomiting. Physical examination shows severe right upper quadrant tenderness. Laboratory studies include serum bilirubin of 1.0 mg/dL, AST of 25 U/L, ALT of 35 U/L, alkaline phosphatase of 220 U/L (high), WBC of 14,000/μL, and amylase of 95 U/L (normal). An ultrasound examination of the abdomen reveals a normal-appearing liver and bile duct and thickening of the wall of the gallbladder. Which of the following is the most likely diagnosis? (A) Acute cholecystitis (B) Acute pancreatitis (C) Adenocarcinoma of the gallbladder (D) Adenocarcinoma of the pancreas (E) Primary biliary cirrhosis

The answer is A: Acute cholecystitis. Acute cholecystitis refers to diffuse infl ammation of the gallbladder, usually secondary to obstruction of the gallbladder outlet. Approximately 90% of cases of acute cholecystitis are secondary to gallstones (cholelithiasis). In patients with acute cholecystitis, the external surface of the gallbladder is intensely congested and often layered with a fi brinous exudate. Acute pancreatitis (choice B) is incorrect because the serum amylase level in this patient is normal. Unsuspected gallbladder cancer (choice C) may be discovered in cholecystectomy specimens, but such an occurrence is uncommon. Pancreatic carcinoma (choice D) often presents as painless jaundice. Diagnosis: Acute cholecystitis

A previously healthy 38-year-old man complains of yellow discoloration of his eyes, abdominal pain, and low-grade fever of 1-month duration. Physical examination demonstrates a distended abdomen, right upper quadrant tenderness, and a palpable liver edge 2 cm below the right costal margin. Total serum bilirubin is 7.4 mg/dL. Serum levels of AST and ALT are elevated (229 and 495 U/L, respectively). The prothrombin time is prolonged (18 seconds). A liver biopsy is shown in the image. The arrows point to Councilman bodies. The pathologic fi ndings are indicative of which of the following liver diseases? (A) Acute viral hepatitis (B) Alcoholic cirrhosis (C) Cardiac cirrhosis (D) Hemochromatosis (E) Primary biliary cirrhosis

The answer is A: Acute viral hepatitis. Pathologic changes of acute viral hepatitis include disarray of liver cell plates, ballooning degeneration of hepatocytes, intracellular and extracellular bile stasis, apoptotic (Councilman) bodies, and mononuclear infl ammatory cell infi ltrates. Histologic manifestations are similar in acute hepatitis A, B, and C. Liver damage in acute viral hepatitis is refl ected in elevations of serum transaminases and hyperbilirubinemia. Severe liver damage leads to impaired production of serum proteins, including prothrombin and other coagulation factors. The other choices are examples of chronic liver disease. Diagnosis: Viral hepatitis, acute

The patient described in Question 39 is at increased risk of developing which of the following complications? (A) Adenocarcinoma (B) Fistula (C) Granulomatous lymphadenitis (D) Transmural infl ammation (E) Volvulus

The answer is A: Adenocarcinoma. Patients with longstanding ulcerative colitis have a higher risk of developing colorectal cancer (adenocarcinoma) than does the general population. This risk is related to the extent of colorectal involvement and the duration of the infl ammatory process. Thus, people with involvement of the entire colon are at the greatest risk of developing colorectal cancer. Intestinal fi stula (choice B) is a complication of Crohn disease. Ulcerations in ulcerative colitis are largely confi ned to the mucosa (not transmural, choice D). Diagnosis: Ulcerative colitis

A 59-year-old man complains of weakness and 5 kg (11 lb) weight loss during the past month. An abdominal CT scan suggests metastatic cancer involving the liver and the retroperitoneum. A CT-guided biopsy displays a poorly- differentiated neoplasm. Electron microscopy of the biopsy is shown. Which of the following organs is the most likely primary site for this patient's malignant neoplasm? (A) Adrenals (B) Kidneys (C) Prostate (D) Testes (E) Thyroid

The answer is A: Adrenals. The electron micrograph of this patient's metastatic cancer shows cells with granules with dense cores. Membrane-bound dense core granules are characteristic of endocrine and neuroendocrine cells such as catecholamineproducing cells and tumors of the adrenal medulla (e.g., pheochromocytoma). None of the other organs is composed of endocrine or neuroendocrine cells. Electron microscopy can be used as an adjunct for pathologic diagnosis when other markers of cellular differentiation are lacking. Carcinomas often exhibit desmosomes and specialized junctional complexes. The presence of melanosomes signifi es a melanoma. Diagnosis: Pheochromocytoma

A 49-year-old woman presents with a 1-month history of yellow discoloration of her eyes, abdominal pain, malaise, weight loss, and low-grade fever (38.4°C, 101°F). Physical examination shows a distended abdomen with right upper quadrant tenderness and a palpable liver 2 cm below the right costal margin. Laboratory studies reveal decreased serum albumin (2.6 g/dL), elevated serum AST (225 U/L) and ALT (150 U/L), and increased alkaline phosphatase (210 U/L). The prothrombin time is prolonged (15 seconds). A moderate leukocytosis (13,500/μL, 80% neutrophils) is observed. A liver biopsy is shown in the image. These pathologic fi ndings are most commonly associated with which of the following liver diseases? (A) Alcoholic hepatitis (B) Chronic hepatitis B (C) Chronic hepatitis C (D) Hemochromatosis (E) Primary biliary cirrhosis

The answer is A: Alcoholic hepatitis. Acute alcoholic hepatitis is characterized by hepatic steatosis and hydropic swelling of hepatocytes, focal hepatocellular necrosis, neutrophilic infi ltration, and cytoplasmic hyaline inclusions within the hepatocytes (Mallory bodies), which represent precipitated intermediate fi lament proteins. Clinically, alcoholic hepatitis presents with malaise and anorexia, fever, right upper quadrant pain, and jaundice. Mallory hyaline is seen in patients with primary biliary cirrhosis, but the other complihistologic fi ndings in this patient's liver biopsy (e.g., hepatic steatosis, hydropic swelling of hepatocytes, focal hepatocellular necrosis, and neutrophilic infi ltration) are not features of primary biliary cirrhosis (choice E). Mallory bodies are rare in the other choices. Diagnosis: Alcoholic hepatitis

A 30-year-old man from Mexico presents with a 3-week history of constant pain in his upper right quadrant and epigastrium and persistent cough. The patient also reports a recent history of nausea, vomiting, and bloody diarrhea. Physical examination shows hepatomegaly and tenderness over the right upper quadrant. A liver biopsy displays fi broblastic proliferation and trophozoites (shown in the image). Which of the following is the most likely diagnosis? (A) Amebic liver abscess (B) Cystic hydatid disease (C) Hepatic malaria (D) Pyogenic liver abscess (E) Weil disease

The answer is A: Amebic liver abscess. Amebic liver abscess is the most common form of extraintestinal amebiasis, although fewer than 30% of patients have a history of antecedent intestinal amebiasis. The male-to-female ratio is 10:1, and the disease is rare in children. Amebic liver abscess appears with an abrupt onset of fever and dull aching abdominal pain in the right upper quadrant or epigastrium, usually lasting less than 10 days. Jaundice is unusual. The diagnosis is usually made by radiologic or ultrasound demonstration of the liver abscess, in conjunction with serologic testing for antibodies to Entamoeba histolytica. Cystic hydatid disease (choice B) is caused by infection with Echinococcus granulosus and is characterized by cyst formation in the liver over several years. Hepatic malaria (choice C) causes hepatomegaly secondary to hypertrophy and hyperplasia of Kupffer cells. Weil disease (choice E) is caused by infections with Leptospira spirochetes. Diagnosis: Amebic liver abscess

A 6-month-old girl is brought to the physician by her parents, who noticed gradual enlargement of the child's abdomen. Physical examination reveals massive hepatomegaly. Laboratory studies show normal serum levels of albumin, bilirubin, and hepatic enzymes. Liver biopsy discloses enlarged hepatocytes with PAS-positive inclusions. Laboratory studies reveal the presence of amylopectin. Deposits of this abnormal glycogen are also found in a biopsy of skeletal muscle. Which of the following is the appropriate diagnosis? (A) Andersen disease (B) Fabry disease (C) Gaucher disease (D) Krabbe disease (E) Metachromatic leukodystrophy

The answer is A: Andersen disease. Andersen disease, which is also known as glycogen storage disease type IV, is an autosomal recessive genetic disease caused by defi ciency of a glycogen- branching enzyme. This enzyme defi ciency results in the accumulation of abnormal glycogen (amylopectin) in the liver, muscle, and other tissues. In most affected persons, the condition becomes evident in the fi rst months of life. Clinical features of Andersen disease include failure to thrive and hepatosplenomegaly. The disease course is typically characterized by progressive hepatic fi brosis leading to cirrhosis and liver failure. None of the other choices are glycogen storage diseases. Diagnosis: Andersen disease

A 1-month-old infant is brought to the physician by her parents. She has had repeated bouts of bilious vomiting over the past month and cannot be fed adequately. She is in the 10th percentile for weight and the 50th percentile for length. An upper GI series discloses marked narrowing of the midportion of the duodenum. What is the most likely cause of this infant's GI obstruction? (A) Annular pancreas (B) Duodenal polyp (C) Islet cell adenoma (D) Pancreatic pseudocyst (E) Pyloric stenosis

The answer is A: Annular pancreas. Annular pancreas is a congenital condition in which the head of the pancreas surrounds the second portion of the duodenum. The anomaly may be associated with duodenal atresia. Infants present with feeding disorders and growth retardation. Pyloric stenosis (choice E) involves the gastric outlet. Duodenal polyp (choice B) does not occur in infants. Diagnosis: Annular pancreas

A 42-year-old obese woman (BMI = 32 kg/m2) presents with severe abdominal pain that radiates to the back. There is no history of alcohol or drug abuse. The blood pressure is 90/45 mm Hg, respirations are 32 per minute, and pulse is 100 per minute. Physical examination shows abdominal tenderness, guarding, and rigidity. An X-ray fi lm of the chest shows a left pleural effusion. Laboratory studies reveal elevated serum amylase (850 U/L) and lipase (675 U/L), and hypocalcemia (7.8 mg/dL). Which of the following is the most likely diagnosis? (A) Acute cholecystitis (B) Acute pancreatitis (C) Alcoholic hepatitis (D) Chronic calcifying pancreatitis (E) Dissecting aortic aneurysm

The answer is B: Acute pancreatitis. Acute pancreatitis is defi ned as an infl ammatory condition of the exocrine pancreas that results from injury to acinar cells. The disease presents with a spectrum of signs and symptoms. Severe forms are characterized by the sudden onset of abdominal pain, often accompanied by signs of shock (hypotension, tachypnea, and tachycardia). The release of amylase and lipase from the injured pancreas into the serum provides a sensitive marker for monitoring injury to acinar cells. Left pleural effusion is a common fi nding in patients with acute pancreatitis due to local irritation below the diaphragm. The other choices do not feature increases in serum amylase and lipase. Diagnosis: Pancreatitis, acute

Which of the following is most likely associated with the pathogenesis of the condition of the patient described in Question 2? (A) Carcinoid syndrome (B) Cholelithiasis (C) Insulinoma (D) Pancreatic adenocarcinoma (E) Portal hypertension

The answer is B: Cholelithiasis. Some 45% of all patients with acute pancreatitis also have cholelithiasis, and the risk of developing acute pancreatitis in patients with gallstones is 25 times higher than that in the general population. Chronic alcoholism accounts for approximately one third of the cases of acute pancreatitis. Other causes include obstruction of the pancreatic duct by gallstones, intake of drugs, and viral infections. The other choices do not cause acute pancreatitis. Diagnosis: Pancreatitis, acute; cholelithiasis

A 60-year-old alcoholic man presents with a 6-month history of recurrent epigastric pain, progressive weight loss, and foulsmelling diarrhea. The abdominal pain is now almost constant and intractable. An X-ray fi lm of the abdomen reveals multiple areas of calcifi cation in the mid-abdomen. Which of the following is the most likely diagnosis? (A) Carcinoid syndrome (B) Chronic pancreatitis (C) Crohn disease (D) Insulinoma (E) Miliary tuberculosis

The answer is B: Chronic pancreatitis. Chronic pancreatitis is characterized by the progressive destruction of the pancreas, with accompanying irregular fi brosis and chronic infl ammation. Calcifi cation and intraductal calculi often develop. Pancreatic insuffi ciency results in malabsorption syndrome. Chronic pancreatitis is most commonly seen in patients with a history of alcohol abuse (70% of cases). The other choices are not associated with pancreatic calcifi cations. Although islets may be affected by chronic pancreatitis, hypoglycemia is an uncommon and late feature of the disease. Diagnosis: Pancreatitis, chronic

A 3-week-old boy is brought to the physician by his parents, who report that he vomits forcefully immediately after nursing. Physical examination reveals an "olive-like" palpable mass and visible peristaltic movements within the infant's abdomen. What is the most likely cause of projectile vomiting in this infant? (A) Appendicitis (B) Congenital pyloric stenosis (C) Hirschsprung disease (D) Meconium ileus (E) Tracheoesophageal fi stula

The answer is B: Congenital pyloric stenosis. Congenital pyloric stenosis is a concentric enlargement of the pyloric canal that obstructs the outlet of the stomach. The disorder is the most common indication for abdominal surgery in the fi rst 6 months of life. Congenital pyloric stenosis has a familial tendency, and the condition is more common in identical twins than in fraternal ones. The only consistent microscopic abnormality is hypertrophy of the circular muscle coat. Projectile vomiting is not characteristic of the other choices, particularly in neonates. Diagnosis: Congenital pyloric stenosis

A 35-year-old man complains of severe acute periumbilical pain that radiates to his back and nausea. The patient recently had a heart transplant for idiopathic cardiomyopathy and is taking azathioprine for immunosuppression. Physical examination reveals bruising of both fl anks. Blood pressure is 120/70 mm Hg, pulse rate 100 per minute, and temperature 37.8°C (100°F). Laboratory studies show elevated serum levels of amylase (950 U/L) and lipase (780 U/L), normal levels of serum calcium, and a normal serum lipid profi le. The patient expires, and the pancreas is examined at autopsy (shown in the image). Which of the following is the most likely underlying cause of these pathologic fi ndings? (A) Acute ischemia (B) Drug-induced pancreatitis (C) Graft-versus-host reaction (D) Hypercalcemia (E) Hyperlipidemia

The answer is B: Drug-induced pancreatitis. Acute pancreatitis may be encountered in patients taking immunosuppressive drugs, antineoplastic agents, sulfonamides, and diuretics. Severe cases of acute pancreatitis cause retroperitoneal hemorrhage, which can track to the fl ank and periumbilical region (see photograph). The other choices may induce pancreatitis but are exceedingly unlikely in this clinical setting. Diagnosis: Pancreatitis, acute

A 22-year-old woman from India presents with a 1-week history of fever, malaise, and nausea. The patient is 6 months pregnant. Physical examination reveals jaundice and right upper quadrant pain. Results of laboratory studies include serum bilirubin of 5.2 mg/dL (60% conjugated), AST of 400 U/L, ALT of 392 U/L, alkaline phosphatase of 70 U/L, anti-HAV antibodies negative, HBsAg negative, and IgM anti- hepatitis E virus (anti-HEV) antibodies positive. The patient is at high risk for which of the following? (A) Diabetes mellitus (B) Fulminant liver failure (C) Pulmonary thromboembolism (D) Renal failure (E) Sclerosing cholangitis

The answer is B: Fulminant liver failure. The patient suffers from hepatitis E infection based upon antibody titers against the virus. HEV is an enteric RNA virus transmitted by the fecal-oral route. It accounts for more than half of cases of acute viral hepatitis in young to middle-aged persons in poor regions of the world. HEV is endemic in parts of Asia and South America, where there is poor sanitation. Like hepatitis A, hepatitis E usually presents as an acute self-limited illness. The average incubation period is 35 to 40 days. The symptoms (jaundice, fever, and arthralgia) usually resolve within 6 weeks. Although overall mortality rates range from 1% to 12%, the disease is especially dangerous in pregnant women due to fulminant hepatic failure, with mortality rates as high as 20% to 40%. None of the other choices are directly associated with HEV infection. Diagnosis: Hepatitis E, acute

A 65-year-old man is brought to the emergency room in a disoriented state. The patient has an odor of alcohol on his breath. Physical examination reveals palmar erythema, diffuse spider angiomata on the upper trunk and face, and gynecomastia. A liver biopsy shows micronodular cirrhosis, massive steatosis, and Mallory hyaline. Serum levels of ammonia are elevated. Which of the following is the most likely underlying cause of gynecomastia in this patient? (A) Hyperbilirubinemia (B) Hyperestrogenism (C) Hypersensitivity vasculitis (D) Hypoalbuminemia (E) Ketoacidosis

The answer is B: Hyperestrogenism. This patient is in hepatic coma and likely suffers from cirrhosis. Chronic liver failure in men leads to feminization, characterized by gynecomastia, female body habitus, and a change in pubic hair distribution. Other features of chronic liver disease include spider angiomata and palmar erythema. These fi ndings are attributed to reduced hepatic catabolism of estrogens (i.e., hyperestrogenism). The other choices are unrelated to feminization. Diagnosis: Alcoholic liver disease

A 69-year-old woman arrives in the emergency room complaining of weakness, abdominal pain, and a 9 kg (20 lb) weight loss during the past month. Physical examination reveals jaundice, conspicuous hepatomegaly, and ascites. The patient expires, and a section of liver is examined at autopsy (shown in the image). Which of the following is the most likely diagnosis? (A) Hemangiosarcoma of the liver (B) Metastatic carcinoma of the liver (C) Miliary tuberculosis (D) Primary hepatocellular carcinoma (E) Sarcoidosis

The answer is B: Metastatic carcinoma of the liver. Liver metastasis is the most common cause of massive hepatomegaly and the most common tumor of the liver. The liver is involved in one third of all metastatic cancers, including half of those of the gastrointestinal tract, breast, and lungs. Other tumors that characteristically metastasize to the liver are pancreatic carcinoma and malignant melanoma, although any cancer may fi nd its way to the liver. Although hemangiosarcomas of the liver (choice A) are frequently multifocal, the tumors are hemorrhagic. Primary hepatocellular carcinoma (choice D) is incorrect because it usually shows a solitary, poorly circumscribed mass, generally in the background of cirrhosis. Miliary tuberculosis (choice C) and sarcoidosis (choice E) feature mm-sized infl ammatory nodules (minute granulomas). Diagnosis: Metastatic carcinoma of the liver

A 60-year-old man is found in a state of disorientation and is brought to the emergency room in a comatose state. He lived alone, ate poorly, and drank large amounts of hard liquor. Physical examination reveals an emaciated man with a distended abdomen, jaundice, ascites, and a slightly enlarged liver and spleen. A liver biopsy is shown in the image. What blood test would confi rm a diagnosis of hepatic coma? (A) Alanine aminotransferase (B) Alkaline phosphatase (C) Ammonia (D) Bilirubin (E) Urea nitrogen

The answer is C: Ammonia. The photomicrograph shows cirrhosis, with regenerative nodules of liver cells surrounded by fi brous septa. Hepatic encephalopathy, a syndrome frequently observed in patients with cirrhosis of the liver, is characterized by personality changes, intellectual impairment, and a depressed level of consciousness. The development of hepatic encephalopathy is caused by increased serum concentrations of neurotoxic substances, among which is ammonia. Choices A, B, and D are elevated in a variety of liver diseases but are unrelated to hepatic encephalopathy. Blood urea nitrogen (choice E) is used to assess kidney function. Diagnosis: Hepatic encephalopathy, alcoholic cirrhosis

Which of the following is a possible cause of liver disease in the patient described in Question 45? (A) Acute hepatitis A (B) Annular pancreas (C) Biliary atresia (D) Cholelithiasis (E) Cystic fi brosis

The answer is C: Biliary atresia. In about half of all cases of neonatal hepatitis, the cause is discernible, and about 30% of cases are assigned to α1-antitrypsin defi ciency alone. Most of the other cases with known causes can be attributed to chromosomal abnormalities or intrauterine infections. Another cause of neonatal hepatitis is biliary atresia, most often presenting with the extrahepatic form. Untreated biliary obstruction causes progressive fi brosis and may result in secondary biliary cirrhosis. The other choices are not associated with neonatal hepatitis. Diagnosis: Biliary atresia

Which of the following is the most important risk factor for the neoplasm arising in the patient described in Question 10? (A) Alcohol abuse (B) Cholelithiasis (C) Cigarette smoking (D) Diabetes mellitus type 1 (E) High-fat diet

The answer is C: Cigarette smoking. Cigarette smoking is associated with a fi vefold increased risk for adenocarcinoma of the pancreas. Cholelithiasis (choice B) and alcohol abuse (choice A) are associated with pancreatitis, not pancreatic adenocarcinoma. Diagnosis: Pancreatic adenocarcinoma

A 22-year-old man presents with a 3-week history of yellow skin and sclerae but is otherwise asymptomatic. He recalls a similar episode 2 years previously. Occasionally, he has noticed dark-colored urine. The serum bilirubin is 4.4 mg/dL, mostly in the conjugated form. Serum AST and ALT levels are normal. A liver biopsy is shown in the image. Which of the following is the most likely diagnosis? (A) α1-Antitrypsin defi ciency (B) Crigler-Najjar syndrome (C) Dubin-Johnson syndrome (D) Gilbert syndrome (E) Wilson disease

The answer is C: Dubin-Johnson syndrome. Dubin-Johnson syndrome is a benign autosomal recessive disease that is characterized by chronic conjugated hyperbilirubinemia and conspicuous melanin-like pigment deposition in the liver. The disease is linked to mutations that result in the complete absence of multidrug resistance protein 2 in hepatocytes. The microscopic appearance of the liver in patients with Dubin- Johnson syndrome is normal, except for the accumulation of coarse, iron-free, dark brown granules in hepatocytes and Kupffer cells. Although all of the other choices are genetic disorders that affect the liver, none present with the clinicopathologic fi ndings seen in this case. Diagnosis: Dubin-Johnson syndrome

A 35-year-old woman presents with 6-month history of skin rash and fatigue. Physical examination shows pallor and a necrotizing erythematous skin rash of her lower body. Laboratory studies reveal mild anemia and fasting blood glucose of 160 mg/dL. A CT scan of the abdomen demonstrates a 2-cm mass in the pancreas. Which of the following is the most likely diagnosis? (A) Carcinoid tumor (B) Gastrinoma (C) Glucagonoma (D) Insulinoma (E) Pancreatic polypeptide-secreting tumor

The answer is C: Glucagonoma. Necrotizing migratory erythema develops in association with the hypersecretion of glucagon by alpha cell-containing tumors (glucagonomas). These patients also have mild hyperglycemia and anemia. The other choices do not present with these clinical signs and symptoms. Diagnosis: Glucagonoma

A 2-year-old girl with a history of chronic constipation since birth is brought to the emergency room because of nausea and vomiting. Physical examination shows marked abdominal distension. Abdominal radiography reveals distended bowel loops with a paucity of air in the rectum. A rectal biopsy shows an absence of ganglion cells. Which of the following is the most likely diagnosis? (A) Acquired megacolon (B) Anorectal stenosis (C) Hirschsprung disease (D) Imperforate anus (E) Rectal atresia

The answer is C: Hirschsprung disease. Hirschsprung disease, also referred to as congenital megacolon, results from a congenital defect in the innervation of the large intestine, usually in the rectum. Severe chronic constipation is typical. Marked dilation of the colon occurs proximal to the stenotic rectum, with clinical signs of intestinal obstruction. The other choices are not associated with loss of ganglion cells. Diagnosis: Hirschsprung disease

A 65-year-old man with a history of acromegaly complains of recurrent epigastric pain and dark-colored tarry stools. Laboratory studies reveal moderate hypercalcemia, hyperlipidemia, and elevated serum levels of PTH and gastrin. Serum glucose is within normal limits. Abdominal ultrasound shows a mass in the tail of the pancreas. Which of the following is the most likely diagnosis? (A) Glucagonoma (B) Insulinoma (C) Multiple endocrine neoplasia type 1 (D) Multiple endocrine neoplasia type 2 (E) Pancreatic carcinoid

The answer is C: Multiple endocrine neoplasia type 1. The patient exhibits signs and symptoms of MEN-1 (Wermer syndrome), including adenoma of the pituitary (acromegaly), hyperplasia or adenoma of the parathyroids (hypercalcemia), and adenoma of the endocrine pancreas (gastrinoma). Gastrin-producing tumors of the pancreas may produce Zollinger-Ellison syndrome, characterized by intractable peptic ulcers. MEN-1 is caused by mutations in the MEN1 tumor suppressor gene. MEN-2A (Sipple syndrome, choice D) features medullary thyroid carcinoma and pheochromocytoma, and is associated with RET protooncogene mutations. Diagnosis: Zollinger-Ellison syndrome, multiple endocrine neoplasia

A 4-week-old infant has been evaluated for jaundice and hepatomegaly since birth. Laboratory studies reveal markedly elevated serum levels of bilirubin and alkaline phosphatase and high serum levels of AST and ALT. A liver biopsy is shown in the image. Which of the following is the most likely diagnosis? (A) Autoimmune hepatitis (B) Dubin-Johnson syndrome (C) Neonatal hepatitis (D) Reye syndrome (E) Sclerosing cholangitis

The answer is C: Neonatal hepatitis. Histologic features of neonatal hepatitis include prolonged cholestasis, infl ammation, and cell injury. Giant cell transformation of hepatocytes is common. These cells contain as many as 40 nuclei and may appear detached from other cells in the liver plate. Most infants with uncomplicated neonatal hepatitis eventually recover. The other choices do not affl ict newborns and do not feature multinucleation. Diagnosis: Neonatal hepatitis

A 20-year-old woman complains of intermittent, colicky abdominal pain, fi ne tremors of her hands, excess sweating, and a general feeling of restlessness. Laboratory studies reveal an inherited defect in the biosynthesis of heme. This patient's genetic disease is most likely caused by defi ciency of which of the following liver enzymes? (A) Alanine aminotransferase (B) Alkaline phosphatase (C) Porphobilinogen deaminase (D) Uridine diphosphate glucuronyl transferase (E) Uroporphyrinogen decarboxylase

The answer is C: Porphobilinogen deaminase. Acute intermittent porphyria is the most common genetic porphyria. This autosomal dominant genetic disease is caused by a defi ciency of porphobilinogen deaminase activity in the liver. Clinical symptoms include colicky abdominal pain and neuropsychiatric symptoms. Choices A, B, and D are not involved in heme biosynthesis. Defi ciency of uroporphyrinogen decarboxylase (choice E) causes a chronic hepatic porphyria that typically presents with cutaneous photosensitivity and iron overload in the middle-aged or elderly. Diagnosis: Porphyria, acute intermittent

A 59-year-old man complains of vague abdominal pain, intermittent fever, and a 9-kg (20-lb) weight loss over the past 8 months. His past medical history is signifi cant for drug abuse, although he claims to be drug free for the past 10 years. On physical examination, the patient shows diffuse abdominal tenderness, hepatomegaly, and mild jaundice. Serologic tests for antibodies to the hepatitis B core antigen (IgG anti-HBcAg) and surface antigen (HBsAg) were negative elsewhere. A CT scan of the abdomen reveals a diffusely nodular liver, with a dominant mass measuring 5 cm in diameter. A liver biopsy shows neoplastic hepatocytes. Which of the following is the most likely underlying cause of this patient's neoplasm? (A) α1-Antitrypsin defi ciency (B) Autoimmune hepatitis (C) Hemochromatosis (D) Hepatitis C virus (E) Primary biliary cirrhosis

The answer is D: Hepatitis C virus (HCV). Although, hepatitis C has a lower global prevalence than hepatitis B, the former is associated with most cases of hepatocellular carcinoma in the United States. Most patients with HCV who develop hepatocellular carcinoma have evidence of chronic liver disease and cirrhosis. α1-Antitrypsin (choice A) defi ciency and hemochromatosis (choice C) are both associated with an elevated risk of hepatocellular carcinoma, but they are far less common than hepatitis C. Primary biliary cirrhosis (choice E) does not lead to hepatocellular carcinoma. Diagnosis: Hepatocellular carcinoma

A 40-year-old woman comes to the physician with a 6-week history of episodic hunger and fainting spells. She is currently seeing a psychiatrist because she is irritable and quarreling with her family. Laboratory studies show a serum glucose concentration of 35 mg/dL. A CT scan of the abdomen demonstrates a 1.5-cm mass in the pancreas. The gross appearance of the bisected tumor is shown in the image. What is the most likely diagnosis? (A) Adenocarcinoma (B) Gastrinoma (C) Glucagonoma (D) Insulinoma (E) Somatostatinoma

The answer is D: Insulinoma. Insulinoma is the most common islet cell tumor. These tumors of the endocrine pancreas are low-grade malignant neoplasms. Insulinomas secrete insulin and cause hypoglycemia. Symptoms of hypoglycemia include hunger, sweating, irritability, epileptic seizures, and coma. Infusion of glucose alleviates these symptoms. The other tumors do not cause marked hypoglycemia. Patients with a glucagonoma (choice C) typically present with necrotizing migratory erythema, mild hyperglycemia, and anemia. Patients with a somatostatinoma (choice E) typically present with mild diabetes mellitus, gallstones, steatorrhea, and hypochlorhydria. Diagnosis: Insulinoma

A 60-year-old man presents with a 3-week history of weight loss, vague abdominal pain, and progressive yellowing of his skin and sclerae. He also reports the recent onset of intermittent pain in the upper and lower extremities. Laboratory studies show a serum bilirubin level of 15 mg/dL, mostly in the conjugated form. A CT scan of the abdomen reveals a mass in the head of the pancreas. The patient develops sudden shortness of breath and is diagnosed with pulmonary thromboembolism. Which of the following is the most likely cause of thromboembolism in this patient? (A) Adenocarcinoma of the ampulla of Vater (B) Gastrinoma of the pancreas (C) Insulinoma of the pancreas (D) Pancreatic adenocarcinoma (E) Pancreatic pseudocyst

The answer is D: Pancreatic adenocarcinoma. Adenocarcinoma is the most common malignant tumor of the pancreas. Although it accounts for only 3% of all cancers in the United States, it is the fourth leading cause of cancer death in men and the fi fth leading cause of cancer death in women. Migratory thrombophlebitis, which is also referred to as Trousseau syndrome, may accompany adenocarcinoma of the pancreas as well as other malignancies. The cause of migratory thrombophlebitis is not entirely understood, but it is thought that the tumor releases thrombogenic substances into the circulation (e.g., serine proteases) that initiate the coagulation cascade. The CT scan excludes adenocarcinoma of the ampulla of Vater (choice A). Endocrine tumors of the pancreas (choices B and C) are not expected to induce Trousseau syndrome. Diagnosis: Pancreatic adenocarcinoma

A 63-year-old woman presents with a 6-month history of recurrent epigastric pain and nausea. Abdominal ultrasound reveals a 13-mm hypoechoic lesion in the tail of the pancreas. Physical examination shows fl ushing of the face, periorbital edema, and hypotension (blood pressure = 90/50 mm Hg). Laboratory studies disclose normal serum levels of gastrin, amylase, insulin, and vasoactive intestinal polypeptide. Urinalysis demonstrates elevated levels of metanephrines (10 mg per 24 hours). Which of the following is the most likely diagnosis? (A) Adenocarcinoma of pancreas (B) Glucagonoma (C) Insulinoma (D) Pancreatic carcinoid (E) Somatostatinoma

The answer is D: Pancreatic carcinoid. Carcinoid tumors of the pancreas are rare malignant neoplasms that closely resemble intestinal carcinoids. When confi ned to the pancreas, they may induce the so-called atypical carcinoid syndrome, which is associated with severe facial fl ushing, hypotension, periorbital edema, and tearing. Hepatic metastases cause the full blown carcinoid syndrome. Adenocarcinoma of the pancreas (choice A) does not produce hormones. The other choices lead to other endocrine syndromes. Diagnosis: Carcinoid tumor

A 45-year-old woman complains of right upper quadrant abdominal pain, weight loss, dry mouth, increased urine production, and foul-smelling fatty stools. She has a recent history of mild diabetes mellitus. Abdominal ultrasound examination reveals gallstones and a solitary 1.5-cm mass in the pancreas. Which of the following hormones would most likely be elevated in the blood of this patient? (A) Calcitonin (B) Gastrin (C) Insulin (D) Somatostatin (E) Vasoactive intestinal polypeptide

The answer is D: Somatostatin. Delta cell tumors (somatostatinomas) produce a syndrome consisting of mild diabetes mellitus, gallstones, steatorrhea, and hypochlorhydria. These effects result from the inhibitory action of somatostatin on the secretion of hormones by cells of the endocrine pancreas, acinar cells of the pancreas, and certain hormone-secreting cells in the gastrointestinal tract. Somatostatin also inhibits the pituitary release of growth hormone. None of the other choices are associated with mild diabetes or cholelithiasis. Diagnosis: Somatostatinoma, cholelithiasis

Despite best efforts at treatment, the patient described in Question 8 subsequently dies. The gross appearance of the pancreas and liver at autopsy is shown in the image. This patient's tumor most likely arose from which of the following types of cells? (A) Acinar cells (B) Alpha cells (C) Beta cells (D) Delta cells (E) Ductal cells

The answer is E: Ductal cells. The majority of pancreatic carcinomas arise from pancreatic duct epithelium. Acinar cell carcinoma (choice A) is much less common. The other choices represent uncommon islet cell tumors. Diagnosis: Pancreatic adenocarcinoma

A 55-year-old, obese man (BMI = 34 kg/m2) comes to the physician for a routine physical examination. His past medical history is signifi cant for type 2 diabetes mellitus that is controlled by medication and diet. The patient neither drinks nor smokes. Physical examination shows mild hepatomegaly. Laboratory studies reveal normal serum levels of albumin and bilirubin and mildly elevated serum levels of AST and ALT (80 and 100 U/L, respectively). The serum level of alkaline phosphatase is normal (70 U/L), and total serum cholesterol is elevated to 290 mg/dL. The CBC is normal. Abdominal ultrasound reveals diffuse fatty infi ltration of the liver. Which of the following is the most likely diagnosis? (A) Autoimmune hepatitis (B) Cirrhosis of the liver (C) Diabetic ketoacidosis (D) Glycogen storage disease (E) Nonalcoholic fatty liver disease

The answer is E: Nonalcoholic fatty liver disease. Nonalcoholic fatty liver is so named because it closely resembles alcoholic fatty liver. This condition represents a spectrum of liver injuries that initially display steatosis, with or without hepatitis. Nonalcoholic fatty liver not infrequently progresses to bridging fi brosis and cirrhosis of the liver. Risk factors for nonalcoholic fatty liver disease include obesity, type 2 diabetes mellitus, and hyperlipidemia. Choices A, B, and D are not fatty liver diseases. Diabetic ketoacidosis (choice C) may be associated with increased fat in the liver, but the patient clearly does not have this disorder. Diagnosis: Nonalcoholic fatty liver disease

The surgical specimen is shown in the image for the patient described in Question 6. In addition to blood and necrotic debris, which of the following best describes the contents of this cystic lesion? (A) Bile (B) Chylous fl uid (C) Lymph (D) Mucopolysaccharides (E) Pancreatic enzymes

The answer is E: Pancreatic enzymes. Pancreatic pseudocysts are lined by connective tissue and contain blood, necrotic debris, and secreted pancreatic enzymes. Refl ux of bile (choice A) is not characteristic of a pancreatic pseudocyst. The other choices (B, C, and D) may be present in small quantities. Diagnosis: Pancreatic pseudocyst

A 50-year-old woman complains of persistent abdominal pain, anorexia, and abdominal distention. Her past medical history is signifi cant for a previous hospitalization for acute pancreatitis. Physical examination shows jaundice and a nonpulsatile abdominal mass. Laboratory studies reveal normal serum levels of bilirubin, AST, and ALT. A CT scan of the abdomen shows a fl uid-fi lled cavity in the head of the pancreas. What is the most likely diagnosis? (A) Acute hemorrhagic pancreatitis (B) Insulinoma (C) Pancreatic cystadenoma (D) Pancreatic islet cell tumor (E) Pancreatic pseudocyst

The answer is E: Pancreatic pseudocyst. Pancreatic pseudocyst is a late complication of acute pancreatitis, in which necrotic pancreatic tissue is liquefi ed through the action of pancreatic enzymes (e.g., peptidases, lipases, and amylase). The necrotic tissue becomes encapsulated by granulation tissue, which then develops into a fi brous capsule. Pseudocysts may enlarge to compress and even obstruct the duodenum. They may become secondarily infected and form an abscess. Choices B, C, and D are not consequences of acute pancreatitis. Diagnosis: Pancreatic pseudocyst

A 60-year-old man has a 6-month history of abdominal swelling. On a daily basis, he smokes two packs of cigarettes, drinks fi ve cups of coffee, and reports that he consumes 2 sixpacks of beer. Physical examination shows a distended abdomen with a palpable liver 2 cm below the costal margin. A liver biopsy is shown in the image. If this patient becomes abstinent, his liver will most likely do which of the following? (A) Develop hepatocellular carcinoma (B) Progress to cirrhosis (C) Progress to infl ammatory hepatitis (D) Remain unchanged (E) Revert to normal

The answer is E: Revert to normal. Excessive alcohol consumption induces fat accumulation within hepatocytes, enlarging the liver to as much as three times the normal weight. The amount of fat deposited varies with the amount of alcohol consumed, as well as the patient's hormonal status, diet, and other factors. Triglyceride accumulation by itself is not ordinarily damaging, and the condition is fully reversible upon discontinuation of alcohol abuse ( abstinence). Diagnosis: Alcoholic fatty liver

Which of the following fi ndings is most likely to be encountered in the patient described in Question 4? (A) Achlorhydria (B) Hypoglycemia (C) Melena (D) Pernicious anemia (E) Steatorrhea

The answer is E: Steatorrhea. Fat malabsorption in the setting of chronic pancreatitis is most often associated with steatorrhea. In patients with steatorrhea, the fecal matter is foul smelling and fl oats because of a high fat content. Longstanding malabsorptive disease is accompanied by nutritional defi ciency, including weight loss, anemia, osteomalacia, and a tendency to bleed. Hypoglycemia (choice B) is incorrect because loss of pancreatic islet cells would be associated with hyperglycemia. Diagnosis: Pancreatitis, chronic; steatorrhea

A 25-year-old woman is brought to the emergency room with symptoms of acute intestinal obstruction. The patient has an 8-month history of blood-tinged diarrhea and cramping abdominal pain. Her temperature is 38°C (101°F), and respirations are 32 per minute. There is abdominal tenderness to palpation. Laboratory studies show moderate anemia, with serum hemoglobin of 9.3 g/dL. Microscopic examination of the stool reveals numerous RBCs and WBCs. A CT scan of the abdomen shows massive distention of the transverse colon. Which of the following is the most likely underlying cause of this patient's colonic disorder? (A) Adenocarcinoma (B) Carcinoid tumor (C) Crohn disease (D) Pseudomembranous colitis (E) Ulcerative colitis

The answer is E: Ulcerative colitis. Local complications of ulcerative colitis include toxic megacolon, perforation, infl ammatory pseudopolyps, hemorrhage, and adenocarcinoma. The other choices are not associated with the development of toxic megacolon. Diagnosis: Toxic megacolon, ulcerative colitis

A 36-year-old woman complains of a 4-week history of unremitting watery diarrhea. She reports that she is always thirsty and drinks continuously. Laboratory studies show achlorhydria, hypokalemia, and mild acidosis. A CT scan of the abdomen demonstrates a 1.5-cm pancreatic mass. Which of the following is the most likely diagnosis? (A) Carcinoid tumor (B) Gastrinoma (C) Pancreatic polypeptide-secreting tumor (D) Somatostatinoma (E) VIPoma

The answer is E: VIPoma. Intractable diarrhea, hypokalemia, and low levels of chloride in gastric juice constitute the syndrome of "pancreatic cholera." This disorder is secondary to the secretion of vasoactive intestinal polypeptide (VIP) by an islet cell tumor. VIP stimulates adenylyl cyclase activity, which in turn leads to the production of large amounts of cAMP. The latter causes increased secretion of potassium and water into the intestinal lumen. The ensuing diarrhea results in loss of water, amounting to as much as 5 L per day. The other choices do not present with these signs and symptoms. Diagnosis: VIPoma

A 47-year-old man suffers from long-standing peptic ulcer disease, which is largely unresponsive to pharmacologic therapy. Endoscopic examination reveals multiple, nonhealed ulcerations of the duodenum and jejunum. Which of the following is the most likely diagnosis? (A) Carcinoid syndrome (B) Insulinoma (C) Pancreatic adenocarcinoma (D) Verner-Morrison syndrome (E) Zollinger-Ellison syndrome

The answer is E: Zollinger-Ellison syndrome. Zollinger- Ellison syndrome is characterized by intractable gastric hypersecretion, severe peptic ulceration of the duodenum and sometimes the jejunum, and elevated levels of gastrin in blood. The tumor responsible for Zollinger-Ellison syndrome is pancreatic gastrinoma composed of G cells. Gastrinomas are most often located in the pancreas, but they may arise in other parts of the gastrointestinal tract, notably the duodenum. Most gastrinomas are malignant. Carcinoid syndrome (choice A) is a systemic paraneoplastic disease caused by the release of hormones from carcinoid tumors into venous blood. Clinical features of carcinoid tumors (e.g., fl ushing, bronchial wheezing, watery diarrhea, and abdominal colic) are presumably caused by the release of serotonin, bradykinin, and histamine. Diagnosis: Gastrinoma, Zollinger-Ellison syndrome

A portion of the large bowel was removed from a 34-year-old man with a familial disease that affects his gastrointestinal tract. The surgical specimen is shown in the image. This patient most likely carries a germline mutation in which of the following genes? (A) APC (B) C-myc (C) DCC (D) p53 (E) Ras

The answer is A: APC. The photograph shows numerous adenomas of the colon, consistent with familial adenomatous polyposis (FAP), also termed adenomatous polyposis coli (APC). This autosomal dominant inherited disease accounts for about 1% of colorectal cancers. It is characterized by the progressive development of innumerable adenomatous polyps of the colorectum, particularly in the rectosigmoid region. Germline mutations in the APC gene, a putative tumor suppressor gene, are responsible for FAP. Carcinoma of the colon and rectum is inevitable in these patients, and the mean age of onset is 40 years. The DCC gene ("deleted in colon cancer"— choice C) is a putative tumor suppressor gene that is often missing in colorectal cancers. Activating mutations of the ras protooncogene (choice E) occur early in tubular adenomas of the colon. Diagnosis: Adenomatous polyposis coli

A 36-year-old, alcoholic woman presents with a 1-week history of yellow skin and sclerae. She has suffered persistent headaches. Her vital signs are normal. Physical examination reveals jaundice. Laboratory studies disclose markedly elevated levels of AST and ALT (956 and 1,400 U/L, respectively). A few days later, she develops hepatic encephalopathy and renal failure. A liver biopsy shows prominent centrilobular necrosis. Which of the following is the most likely diagnosis? (A) Acetaminophen toxicity (B) Fatty liver of pregnancy (C) Metastatic carcinoma (D) Reye syndrome (E) Wilson disease

The answer is A: Acetaminophen toxicity. Drug toxicity should be suspected in all cases of acute hepatitis. In this case, centrilobular necrosis suggests acetaminophen toxicity. The toxic dose of acetaminophen after a single acute ingestion is in the range of 150 mg/kg in children and 7 g in adults. Acetaminophen is rapidly absorbed from the stomach and small intestine and conjugated in the liver to nontoxic agents, which then are eliminated in the urine. In cases of acute overdose, normal pathways of acetaminophen metabolism become saturated. Excess acetaminophen is then metabolized in the liver via the mixed function oxidase P450 system, yielding oxidative metabolites that cause predictable, hepatocellular necrosis. The centrilobular zones are particularly affected (centrilobular necrosis). Centrilobular necrosis is not seen in the other choices. Reye syndrome (choice D) occurs in children. Fatty liver of pregnancy (choice B) features microvesicular steatosis. Diagnosis: Acetaminophen toxicity, hepatorenal syndrome

A 35-year-old man complains of diffi culty swallowing and a tendency to regurgitate his food. Endoscopy does not reveal any esophageal or gastric abnormalities. Manometric studies of the esophagus show a complete absence of peristalsis, failure of the lower esophageal sphincter to relax upon swallowing, and increased intraesophageal pressure. Which of the following is the most likely diagnosis? (A) Achalasia (B) Barrett esophagus (C) Esophageal stricture (D) Mallory-Weiss syndrome (E) Schatzki ring

The answer is A: Achalasia. Achalasia is characterized by failure of the lower esophageal sphincter to relax in response to swallowing and the absence of peristalsis in the body of the esophagus. As a result of these defects in both the outfl ow tract and the pumping mechanisms of the esophagus, food is retained within the esophagus, and the organ hypertrophies and dilates. Achalasia is associated with a depletion or absence of ganglion cells in the myenteric plexuses, which regulate contraction of the esophagus. In Latin America, achalasia can be a manifestation of Chagas disease, in which the ganglion cells are destroyed by Trypanosoma cruzi. The other choices are usually associated with visible mucosal abnormalities and do not primarily affect peristalsis. Diagnosis: Achalasia

A 50-year-old woman with long-standing rheumatoid arthritis complains of weakness and fatigue. She states that her stools have recently become black after taking a new nonsteroidal anti-infl ammatory drug (NSAID). Gastroscopy shows numerous superfi cial, bleeding mucosal defects. Which of the following is the most likely diagnosis? (A) Acute erosive gastritis (B) Early gastric cancer (C) Helicobacter pylori gastritis (D) Ménétrier disease (E) Peptic ulcer disease

The answer is A: Acute erosive gastritis. Acute hemorrhagic gastritis is characterized by necrosis of the mucosa and is commonly associated with the intake of aspirin, other NSAIDs, alcohol, or ischemic injury. The factor common to all forms of acute hemorrhagic gastritis is thought to be the breakdown of the mucosal barrier, which permits acid-induced injury. Mucosal injury causes bleeding from superfi cial erosions. Defects in the mucosa may extend into the deeper tissues to form an ulcer. The necrosis is accompanied by an acute infl ammatory response and hemorrhage, which may be severe enough to result in exsanguination and hypovolemic shock. The other choices are not associated with the use of NSAIDs. Diagnosis: Acute erosive gastritis

A 20-year-old woman presents with a 2-week history of fever, malaise, and brown-colored urine. She recently visited Mexico. Physical examination reveals jaundice, mild hepatomegaly, and tenderness in the right upper quadrant. The serum bilirubin is 7.8 mg/dL, with 60% in the conjugated form. Serum levels of AST and ALT are markedly elevated (400 and 392 U/L, respectively). Serum albumin and immunoglobulin levels are normal. Serum IgM anti-hepatitis A virus (anti-HAV) is positive. IgG anti-hepatitis B surface antigen (anti-HBsAg) antibodies are positive. Anti-hepatitis C virus antibodies are negative. What is the most likely diagnosis? (A) Acute viral hepatitis A (B) Acute viral hepatitis B (C) Acute viral hepatitis C (D) Autoimmune hepatitis (E) Chronic viral hepatitis B

The answer is A: Acute viral hepatitis A. HAV is an RNA virus that is transmitted by the fecal-oral route and may be contracted by contamination of water and food. Hepatitis A, the disease presented here, never pursues a chronic course, does not have a carrier state, and provides life-long immunity. IgM anti-HAV is identifi ed in acute infections. The presence of serum IgG anti-HBsAg indicates prior exposure to hepatitis B virus but does not refl ect active disease (choice B). Individuals with chronic hepatitis B (choice E) do not have detectable anti- HBsAg in their blood. Acute and chronic hepatitis C (choice C) are ruled out by the negative serology. Serum immunoglobulins are typically increased in patients with autoimmune hepatitis (choice D). Diagnosis: Hepatitis A, acute

A 70-year-old woman presents with diffi culty swallowing and a 9-kg (20-lb) weight loss over the past several months. Endoscopy reveals irregular narrowing of the lower third of the esophagus. A biopsy shows markedly atypical cuboidal cells lining irregular gland-like structures. Which of the following is the most likely diagnosis? (A) Adenocarcinoma (B) Esophageal stricture (C) Leiomyosarcoma (D) Scleroderma (E) Squamous cell carcinoma

The answer is A: Adenocarcinoma. Adenocarcinoma of the esophagus is now more common (60%) in the United States than squamous carcinoma. Adenocarcinoma originates in the glandular metaplasia of Barrett esophagus. Endoscopic surveillance for adenocarcinoma is now commonly done in patients with Barrett esophagus, particularly those with dysplasia. Tumors tend to grow into the lumen of the esophagus The affected region of the esophagus is typically indurated and ulcerated, causing pain and bleeding. The other choices do not exhibit the histologic features described. Diagnosis: Adenocarcinoma of the esophagus

A 59-year-old man complains of progressive weakness. His friends have noticed that he has become pale, and he reports that his stools are tinged with blood. On abdominal palpation, there is fullness in the right lower quadrant. Laboratory studies show iron-defi ciency anemia, with a hemoglobin level of 7.4 g/dL. Stool specimens are positive for occult blood. Colonoscopy reveals an elevated and centrally ulcerated lesion of the sigmoid colon. The biopsy is shown in the image. Which of the following is the most likely diagnosis? (A) Adenocarcinoma (B) Carcinoid tumor (C) Gastrointestinal stromal tumor (D) Lymphoma (E) Mucinous cystadenoma

The answer is A: Adenocarcinoma. Adenocarcinoma of the rectum or sigmoid colon often presents as a circumferential mass narrowing the intestinal lumen. The gross appearance of the colorectal cancer is similar to that seen elsewhere in the gastrointestinal tract. The most important risk factors associated with the development of colonic adenocarcinoma are age, prior colorectal cancer, ulcerative colitis, genetic factors, and perhaps diet. Colorectal cancer invades lymphatic channels and initially involves the lymph nodes immediately underlying the tumor. As the tumor grows, the most common sign is occult blood in feces. Bright red blood more often occurs in distal lesions. In either case, bleeding typically causes iron-defi ciency anemia. Choices B, C, and D are principally lesions of the intestinal wall. Choice E (mucinous cystadenoma) is an ovarian tumor. Diagnosis: Colorectal cancer, adenocarcinoma of colon

A 25-year-old woman presents with persistent bloody diarrhea of 4 weeks' duration. She has experienced severe abdominal cramping for the past 3 days. Her temperature is 38°C (101°F), respirations are 22 per minute, and blood pressure is 120/70 mm Hg. Physical examination reveals abdominal tenderness and mild abdominal distension. Bowel sounds are diminished. Laboratory studies show mild hypochromic, normocytic anemia. Stool cultures are negative for pathogens, and no ova or parasites are detected. A blood test for Clostridium diffi cile toxin is negative. Rectosigmoidoscopy shows hemorrhagic mucosal lesions in the distal colorectal region. A biopsy of the colon reveals crypt abscesses, basal lymphoplasmacytosis and crypt distortion. Which of the following represents the most common extraintestinal manifestation of the colonic disorder in this patient? (A) Arthritis (B) Cystitis (C) Gastritis (D) Pancreatitis (E) Sepsis

The answer is A: Arthritis. The case history is indicative of ulcerative colitis. Arthritis is seen in 25% of patients with ulcerative colitis. Uveitis and skin lesions develop in approximately 10% of patients. The most common cutaneous lesions are erythema nodosum and pyoderma gangrenosum. Liver disease occurs in about 4% of patients, the most common pathologic fi ndings being pericholangitis and fatty liver. The other choices do not represent extraintestinal manifestations of ulcerative colitis. Diagnosis: Ulcerative colitis, arthritis

A 20-year-old woman presents with a 4-week history of dry mouth, fatigue, fever, and yellow sclerae. Physical examination shows mild jaundice and hepatomegaly. Serum total bilirubin is 3.3 mg/dL. Serologic markers for viral hepatitis are negative. The anti-mitochondrial antibody test is negative. A liver biopsy discloses parenchymal and periportal infl ammatory cell infi ltrates composed primarily of lymphocytes and plasma cells. The patient's signs and symptoms abate following 2 months of treatment with steroids. Which of the following is the most likely diagnosis? (A) Autoimmune hepatitis (B) Extrahepatic jaundice (C) Primary biliary cirrhosis (D) Primary sclerosing cholangitis (E) Wilson disease

The answer is A: Autoimmune hepatitis. Autoimmune hepatitis is a type of chronic hepatitis, which is associated with circulating autoantibodies (e.g., antinuclear antibodies) and high levels of serum immunoglobulins. The disease typically affects young women but occasionally affl icts older women and men. It is often accompanied by other autoimmune diseases (e.g., Sjögren syndrome, systemic lupus erythematosus). None of the other choices respond to steroids. Primary biliary cirrhosis (choice C) features anti-mitochondrial antibodies. Primary biliary cirrhosis (choice C) and primary sclerosing cholangitis (choice D) do not manifest the described histologic fi ndings. Diagnosis: Autoimmune hepatitis

A 60-year-old man presents with epigastric pain after meals, with some nausea and vomiting. A burning sensation in the midepigastrium is relieved by antacids and H2 antagonists. Upper endoscopy demonstrates paired ulcers on both walls of the proximal duodenum. Which of the following represents the most common complication of this patient's duodenal disease? (A) Bleeding (B) Malignant transformation (C) Obstruction (D) Perforation (E) Peritonitis

The answer is A: Bleeding. Bleeding is the most common complication of peptic ulcer disease, occurring in about 20% of patients. Chronic blood loss due to occult bleeding is often a feature of peptic ulcers, whereas massive bleeding occurs less often. Perforation (choice D) is a serious complication that occurs in 5% of patients. Perforating ulcers are commonly encountered in the duodenum. Duodenal peptic ulcers do not undergo malignant transformation (choice B). The other choices are uncommon. Diseases associated with peptic ulcers include cirrhosis, chronic renal failure, hereditary endocrine syndromes (MEN-1), α1-antitrypsin defi ciency, and chronic pulmonary disease. Diagnosis: Duodenal ulcer, peptic ulcer disease

A liver biopsy in the patient described in Question 14 would defi nitely show which of the following pathologic changes? (A) Dilated bile ducts and portal infl ammation (B) Fatty liver (C) Nodular regeneration and scarring (D) Periportal necrosis and peripheral cholestasis (E) Scattered single cell necrosis and acidophilic bodies

The answer is C: Nodular regeneration and scarring. In about 15% of alcoholics, hepatocellular necrosis, fi brosis, and regeneration eventually lead to the formation of fi brous septa surrounding hepatocellular nodules, which are features that defi ne cirrhosis. Morphologic changes described in the other choices may be present in cases of alcoholic liver disease but are not directly associated with portal hypertension. Fatty liver (choice B) and Mallory hyaline are associated with alcoholism, but they are not specifi c indicators of cirrhosis. Diagnosis: Alcoholic cirrhosis

A 28-year-old woman presents with a 4-day history of abdominal pain and increasing abdominal girth. She does not drink alcoholic beverages, but smokes a pack of cigarettes a day. Except for oral contraceptives, she takes no medications. Physical examination shows hepatomegaly, ascites, and mild jaundice. A liver biopsy is obtained (shown in the image). Which of the following is the most likely diagnosis? (A) Budd-Chiari syndrome (B) Chronic hepatitis B (C) Extrahepatic cholestasis (D) Primary biliary cirrhosis (E) Secondary biliary cirrhosis

The answer is A: Budd-Chiari syndrome. Budd-Chiari syndrome is a congestive disease of the liver caused by occlusion of the hepatic veins and their tributaries. The principal cause of Budd-Chiari syndrome is thrombosis of the hepatic veins. Intrahepatic venous thrombosis may be associated with increased blood viscosity (as in polycythemia vera or other myeloproliferative disorders) and hypercoagulable states associated with hematologic cancers, certain solid tumors, pregnancy, and paroxysmal nocturnal hemoglobinuria. However, in more than half the cases, the cause of Budd-Chiari syndrome is not apparent. Complete thrombosis of the hepatic veins presents as an acute illness characterized by abdominal pain, enlargement of the liver, ascites, and mild jaundice. Acute hepatic failure and death often occur rapidly. The needle biopsy in this case shows severe centrilobular necrosis and hemorrhage (see photomicrograph). The sinusoids of the central zone are dilated and packed with erythrocytes. The other choices do not present with centrilobular hemorrhage and are not associated with oral contraceptives. Diagnosis: Budd-Chiari syndrome

A 45-year-old woman presents with sudden attacks of wheezing, shortness of breath, and episodic hot fl ashes. She also reports abdominal cramps and diarrhea. Physical examination shows facial redness, pitting edema of the lower legs, and a murmur of tricuspid regurgitation. A 24-hour urine specimen contains elevated levels of 5-hydroxyindoleacetic acid (5-HIAA). A CT scan of the abdomen demonstrates multiple 1- to 2-cm nodules in distal ileum. A small bowel resection is performed (shown in the image). The arrows point to submucosal tumors. Microscopic examination shows nests of cells with round and uniform nuclei. Which of the following is the most likely diagnosis? (A) Carcinoid tumor (B) Mediterranean intestinal lymphoma (C) Mucosa-associated lymphoid tissue (MALT) lymphoma (D) Peutz-Jeghers syndrome (E) Whipple disease

The answer is A: Carcinoid tumor. Carcinoid tumors are low-grade malignant neoplasms composed of neuroendocrine cells, which usually show considerable nuclear uniformity. They are most commonly located in the submucosa of the intestines (appendix, terminal ileum, and rectum). Carcinoids are distinguished from intestinal carcinomas based on their location, histologic features, malignant potential, endocrine activity, and clinical features. Carcinoid syndrome is a systemic paraneoplastic disease caused by the release of hormones from carcinoid tumors into venous blood. Clinical features of carcinoid tumors (e.g., fl ushing, bronchial wheezing, watery diarrhea, and abdominal colic) are presumably caused by the release of serotonin, bradykinin, and histamine. Release of tumor secretions from hepatic metastases leads to the formation of fi brous plaques in the tricuspid and pulmonic valves and may result in tricuspid insuffi ciency or pulmonic stenosis. The other choices are not associated with secretion of 5-HIAA acid or other neuroendocrine peptides. Diagnosis: Carcinoid syndrome

A 40-year-old woman presents with a long history of vague upper abdominal pain and frequent indigestion. Physical examination reveals an obese woman with jaundice and abdominal tenderness. Serum bilirubin is elevated (4.2 mg/dL). There is a mild increase in serum AST and ALT (62 and 57 U/L, respectively) and a moderate increase in alkaline phosphatase (325 U/L). Markers for viral hepatitis are negative. Abdominal ultrasound examination shows echogenic stone-like material within the gallbladder and thickening of the gallbladder wall. An intrahepatic mass is also visualized adjacent to the gallbladder. A cholecystectomy is performed. Histologic examination shows dense fi brosis and glandular structures in the wall of the gallbladder. What is the most likely diagnosis? (A) Carcinoma of the gallbladder (B) Hemangiosarcoma (C) Hepatic adenoma (D) Hepatocellular carcinoma (E) Metastatic carcinoma of the stomach

The answer is A: Carcinoma of the gallbladder. Adenocarcinoma of the gallbladder is an incidental fi nding in 2% of patients who undergo gallbladder surgery due to chronic cholelithiasis. The tumor arises from the mucosal surface epithelium and may cause obstructive jaundice (as in this case) by involvement of the extrahepatic biliary tree. The other choices are not associated with a history of chronic cholecystitis and cholelithiasis and infrequently cause obstructive jaundice. Diagnosis: Carcinoma of the gallbladder

A 4-year-old girl is brought to the physician because her parents noticed that she has been having pale, fatty, foul-smelling stools. The patient is at the 50th percentile for height and 10th percentile for weight. Her symptoms respond dramatically to a gluten-free diet. Which of the following is the most likely diagnosis? (A) Celiac sprue (B) Cystic fi brosis of the pancreas (C) Ménétrier disease (D) Tropical sprue (E) Whipple disease

The answer is A: Celiac sprue. Celiac sprue, which is also referred to as gluten-sensitive enteropathy, is characterized by (1) generalized malabsorption, (2) small intestinal mucosal lesions, and (3) prompt clinical and histopathologic response to the withdrawal of gluten-containing food. Critical factors in the development of celiac sprue include genetic predisposition and gliadin exposure. The hallmark of celiac disease is a fl at mucosa, with blunting of villi, damaged epithelial cells, intraepithelial T cells, and increased plasma cells in the lamina propria. The other choices do not respond to a gluten-free diet. Diagnosis: Celiac sprue

Which of the following is the most important risk factor for development of the esophageal mass identifi ed in the patient described in Question 10? (A) Cigarette smoking (B) Exposure to afl atoxin (C) Herpetic esophagitis (D) Hot and spicy food (E) Refl ux esophagitis

The answer is A: Cigarette smoking. Risk factors for squamous cell carcinoma of the esophagus include chronic alcoholism, tobacco use, diets lacking in fresh fruits, exposure to aniline dyes, chronic esophagitis, and congenital disorders of the esophagus (e.g., Plummer-Vinson syndrome). Cigarette smoking is associated with a 5- to 10-fold increased risk of esophageal cancer, and the number of cigarettes smoked correlates with the presence of dysplasia in the esophageal epithelium. Epidemiologic data suggest that there are additional, as yet unidentifi ed risk factors prevalent in certain geographical regions of the world (China, Iran, and South Africa). Refl ux esophagitis (choice E) leads to Barrett esophagus and adenocarcinoma. Afl atoxin (choice B) is a well-known hepatotoxin linked to the development of hepatocellular carcinoma. Herpetic esophagitis (choice C) frequently occurs in immunocompromised persons but is not associated with the development of carcinoma. Diagnosis: Esophageal cancer

A 15-year-old boy complains of a 2-month history of fatigue, abdominal pain, and yellow eyes and skin. Physical examination shows tremor of his hands, lack of coordination, and mild jaundice. The results of an ophthalmic examination are shown in the image. This patient most likely has an inborn error of metabolism associated with tissue overload of which of the following elements? (A) Copper (B) Iron (C) Lead (D) Magnesium (E) Mercury

The answer is A: Copper. Wilson disease is an autosomal recessive condition in which excess copper can be deposited in the liver and brain. Chronic hepatitis leads to cirrhosis in young people. Ocular lesions, so-called Kayser-Fleischer rings, represent deposition of copper in Descemet membrane in the iris (note peripheral brown color). Extrapyramidal neurologic symptoms (e.g., lack of coordination and tremor) are related to degenerative changes in the corpus striatum. Toxicity of the other elements are associated with other manifestations. Diagnosis: Wilson disease

A 16-year-old girl complains of chronic abdominal distention, fl atulence, and diarrhea after drinking milk. Elimination of milk and other dairy products from the patient's diet relieves these symptoms. This example of malabsorption is caused by a functional defi ciency of which of the following enzymes associated with the intestinal brush border membrane? (A) Disaccharidase (B) Glycogen phosphorylase (C) Hyaluronidase (D) Mannosidase (E) Sphingomyelinase

The answer is A: Disaccharidase. Acquired lactase defi ciency is a widespread disorder of carbohydrate absorption. The symptoms of this disease typically begin in adolescence, when patients complain of fl atulence and diarrhea after the ingestion of dairy products. Lactose is one of the most common disaccharides in dairy products. The intestinal brush border contains disaccharidases that are important for cleavage of lactose to free glucose and galactose for absorption. Congenital lactase defi ciency is rare but may be lethal if not recognized. The other choices do not hydrolyze lactose. Diagnosis: Lactose intolerance

A 30-year-old man presents with a 3-week history of fatigue, occasional fever, yellow skin and sclerae, tenderness below the right costal margin, and dark urine. Physical examination reveals jaundice and mild hepatomegaly. Laboratory studies show elevated serum levels of bilirubin, decreased albumin, and prolonged prothrombin time. Serologic tests disclose antibodies to hepatitis C virus. Which of the following tests is the most accurate method for assessing the extent of liver disease in this patient? (A) Liver biopsy (B) Serum alkaline phosphatase (C) Serum ammonia (D) Serum immunoglobulins (E) Serum transaminases

The answer is A: Liver biopsy. Microscopic examination of a liver biopsy is the best method currently available for assessing the extent of liver disease in a patient with viral hepatitis. The major histologic features of acute viral hepatitis are liver cell injury and infl ammation. Microscopic examination shows ballooning degeneration of liver cells, intracellular and extracellular bile stasis, acidophilic bodies, and a mononuclear cell infi ltrate. Serum alkaline phosphatase and transaminases (choices B and E) are also useful indicators of the severity of liver disease, but do not allow for an assessment of the chronicity or stage of the disease. Serum ammonia (choice C) is used to monitor patients at risk for hepatic encephalopathy and ordinarily refl ects end-stage liver disease. Diagnosis: Hepatitis C, acute

A 32-year-old man presents with a 6-month history of yellow skin and sclerae. Physical examination shows mild jaundice, pitting edema, and ascites. Laboratory studies reveal decreased serum albumin (2.6 g/dL) and increased serum AST and ALT (120 and 140 U/L, respectively). A liver biopsy stained with period acid-Schiff (PAS) reagent and diastase digestion is shown in the image. This patient has which of the following genetic diseases? (A) α1-Antitrypsin defi ciency (B) Glycogen storage disease (C) Hereditary hemochromatosis (D) Hurler syndrome (E) Pompe syndrome

The answer is A: a1-Antitrypsin defi ciency. α1-Antitrypsin defi ciency is the most common genetic cause of liver disease in infants and children and the most frequent genetic disease for which liver transplantation is indicated. The liver may be involved with or without pulmonary disease in the form of emphysema. α1-Antitrypsin defi ciency is characterized by the presence of round-to-oval cytoplasmic globular inclusions of misfolded α1-antitrypsin proteins in hepatocytes. These globules stain red with PAS after removing glycogen with diastase. These inclusions are not featured in the other choices. Diagnosis: α1-Antitrypsin defi ciency

A 60-year-old man presents with an 8-week history of progressive weight loss, nausea, and upper abdominal pain that does not respond to antacids or H2-receptor antagonists. Laboratory studies show iron-defi ciency anemia. Gastroscopy reveals a crater-like, ulcerated lesion in the antrum, with raised, irregular, and indurated margins. The patient undergoes partial gastrectomy and the surgical specimen is shown in the image. Which of the following is the most likely diagnosis? (A) Acute erosive gastritis (B) Adenocarcinoma (C) Curling ulcer (D) Linitis plastica (E) Peptic ulcer disease

The answer is B: Adenocarcinoma. Adenocarcinoma of the stomach accounts for more than 95% of all malignant gastric tumors. Most patients have metastases by the time they are seen for examination. The most frequent initial symptom of gastric cancer is weight loss, usually associated with anorexia and nausea. Most patients complain of epigastric pain—a symptom that mimics benign gastric ulcer disease, and is often relieved by antacids or H2-receptor antagonists. On gross inspection, gastric cancer appears as a polypoid, fungating, or ulcerated mass, or diffuse infi ltration of the stomach wall. This patient has an ulcerating carcinoma. Acute erosive gastritis (choice A) and peptic ulcer disease (choice E) do not typically present with rapid weight loss, and these benign ulcers usually do not have heaped-up (raised), ragged margins. Curling ulcers (choice C) occur in severely burned patients. Diagnosis: Gastric adenocarcinoma

The patient described in Question 4 is at increased risk of developing which of the following diseases of the esophagus? (A) Achalasia (B) Adenocarcinoma (C) Candidiasis (D) Plummer-Vinson syndrome (E) Varices

The answer is B: Adenocarcinoma. Barrett esophagus carries a serious risk of malignant transformation to adenocarcinoma, and the risk correlates with the length of the involved esophagus and the degree of dysplasia. Virtually all esophageal adenocarcinomas arise in the background of the metaplastic epithelium of Barrett esophagus. The symptoms and clinical course of adenocarcinoma of the esophagus are similar to those of squamous cell carcinoma and include dysphagia, pain, and, occasionally, bleeding. None of the other choices refl ect a complication of Barrett esophagus. Diagnosis: Barrett esophagus

A 68-year-old man complains of vague abdominal pain, intermittent fever, and a 20-lb (9-kg) weight loss over the past 6 months. For the past 12 years, he has suffered from chronic hepatitis B. On physical examination, the patient shows diffuse abdominal tenderness, hepatomegaly, and mild jaundice. A CT scan of the abdomen reveals a diffusely nodular liver, with a dominant mass measuring 3 cm in diameter. A needle biopsy is shown in the image. Which of the following serum markers is useful for monitoring the progression of disease in this patient? (A) Alkaline phosphatase (B) Alpha-fetoprotein (C) Anti-HBc antibody (D) Carcinoembryonic antigen (E) Human chorionic gonadotropin

The answer is B: Alpha-fetoprotein (AFP). AFP is a glycoprotein that is normally synthesized in the fetus by the yolk sac, liver, and gastrointestinal tract. In adults, an elevated serum level of AFP is a useful indicator of hepatocellular carcinoma and germ cell tumors of the testis. AFP levels decline rapidly after surgical resection of liver cell cancer or treatment of patients with metastatic germ cell tumors. Alkaline phosphatase (choice A) is a common indicator of hepatobiliary disease. Carcinoembryonic antigen (choice D) is principally used to monitor gastrointestinal cancers. Diagnosis: Hepatocellular carcinoma

A 34-year-old man presents with a 5-month history of weakness and fatigue. There is no history of drug or alcohol abuse. A CBC shows megaloblastic anemia and a normal reticulocyte count Further laboratory studies reveal vitamin B12 defi ciency. Anemia in this patient is most likely caused by which of the following? (A) Acute erosive gastritis (B) Autoimmune gastritis (C) Helicobacter pylori gastritis (D) Ménétrier disease (E) Peptic ulcer disease

The answer is B: Autoimmune gastritis. Autoimmune gastritis refers to chronic, diffuse infl ammatory disease of the stomach that is restricted to the body and fundus and is associated with other autoimmune phenomena. This disorder typically features diffuse atrophic gastritis, antibodies to parietal cells and the intrinsic factor, and increased serum gastrin due to G-cell hyperplasia. Immunologic destruction of parietal cells and antibody targeting of intrinsic factor interfere with intestinal absorption of vitamin B12. As a result, all lineages of bone marrow precursors show asynchronous maturation between the nucleus and cytoplasm (megaloblastic cells), and the peripheral blood displays megaloblastic anemia. Megaloblastic anemia that is caused by malabsorption of vitamin B12, occasioned by a defi ciency of the intrinsic factor, is referred to as "pernicious anemia." The other choices are not causes of pernicious anemia. Diagnosis: Autoimmune atrophic gastritis, pernicious anemia

A 48-year-old woman has a 3-week history of fatigue as well as yellow skin and sclerae. Physical examination is unremarkable except for mild jaundice. The serum bilirubin level is 3.7 mg/dL, mostly in the unconjugated form. Liver function tests including serum AST, ALT, and alkaline phosphatase are normal. The hemoglobin level is 6.0 g/dL. After corticosteroids are administered, the jaundice resolves. Which of the following diseases is the most likely cause of hyperbilirubinemia in this patient? (A) Acute hepatitis B infection (B) Autoimmune hemolytic anemia (C) Gallstone in the common bile duct (D) Primary biliary cirrhosis (E) Primary sclerosing cholangitis

The answer is B: Autoimmune hemolytic anemia. Autoimmune hemolytic anemia is characterized by antibody- mediated erythrocyte destruction and may lead to severe anemia, as in this case. Intravascular hemolysis produces increased serum levels of unconjugated bilirubin, which exceed the capacity of the hepatocyte to conjugate bilirubin. In most cases, the disease is ameliorated by treatment with corticosteroids. The other choices are parenchymal liver diseases that manifest primarily as conjugated hyperbilirubinemia and are unresponsive to steroids. Diagnosis: Autoimmune hemolytic anemia

A 30-year-old man with AIDS complains of severe pain on swallowing. Upper GI endoscopy shows elevated, white plaques on a hyperemic and edematous esophageal mucosa. Which of the following is the most likely diagnosis? (A) Barrett esophagus (B) Candida esophagitis (C) Herpetic esophagitis (D) Refl ux esophagitis (E) Squamous cell carcinoma in situ

The answer is B: Candida esophagitis. Candida esophagitis has become commonplace because of an increasing number of immunocompromised persons. Esophageal candidiasis also occurs in patients with diabetes and in those receiving antibiotic therapy. The pseudomembranes are composed of fungal mycelia, fi brin, and necrotic debris. Involvement of deeper layers of the esophageal wall can lead to disseminated candidiasis, as well as fi brosis, which is sometimes severe enough anemia that is caused by malabsorption of vitamin B12, occasioned by a defi ciency of the intrinsic factor, is referred to as "pernicious anemia." The other choices are not causes of pernicious anemia. Diagnosis: Autoimmune atrophic gastritis, pernicious anemia

A 52-year-old recent immigrant from Vietnam complains of abdominal swelling, weight loss, and upper abdominal pain of 3 weeks in duration. His past medical history includes malaria and infection with the liver fl uke Clonorchis sinensis. The liver is hard to palpation. An abdominal CT scan shows a hypoattenuated mass with lobulated margins in the liver. A biopsy discloses well-differentiated neoplastic glands embedded in a dense fi brous stroma. Which of the following is the most likely diagnosis? (A) Carcinoma of the gallbladder (B) Cholangiocarcinoma (C) Hemangiosarcoma (D) Hepatocellular carcinoma (E) Metastatic colon adenocarcinoma

The answer is B: Cholangiocarcinoma. Carcinoma originates anywhere in the biliary tree, from the large intrahepatic ducts at the porta hepatis to the smallest bile ductules at the periphery of the hepatic lobules. Cholangiocarcinoma arising within the liver is associated with substantial fi brosis and can be confused with metastatic carcinoma and reactive fi brosis. These tumors occur at an increased frequency in persons infected with the liver fl uke C. sinensis, which takes up residence in the biliary tree. Primary sclerosing cholangitis is another risk factor for this cancer. Patients with cholangiocarcinoma have a poor prognosis. The other choices are not associated with a history of C. sinensis infestation. Diagnosis: Cholangiocarcinoma

A 55-year-old woman complains of upper gastrointestinal pain and tarry stools. Upper endoscopy shows a fi rm, smooth, yellowish submucosal ulcerated mass in the stomach. Gastroscopic biopsy reveals spindle cells with vacuolated cytoplasm. The mass is removed, and the surgical specimen is shown in the image. Which of the following is the most likely diagnosis? (A) Gastric adenocarcinoma (B) Gastric lymphoma (C) Gastrointestinal stromal tumor (D) Peptic ulcer (E) Tubular adenoma

The answer is C: Gastrointestinal stromal tumor (GIST). GISTs are derived from the pacemaker cells of Cajal. They include the vast majority of mesenchyme-derived stromal tumors of the entire gastrointestinal tract. Gastric GISTs are usually submucosal and covered by intact mucosa. Microscopically, the tumors show spindle cells with vacuolated cytoplasms. GISTs are considered to be of low malignant potential and are removed surgically. Gastric adenocarcinoma (choice A) does not often dedifferentiate to a spindle cell morphology. Diagnosis: Gastrointestinal stromal tumor

A 40-year-old woman complains of having severe back pain for about 3 months and recurrent fever. Her past medical history is signifi cant for ulcerative colitis. On physical examination, the patient is thin and jaundiced. The liver edge descends 1 cm below the right costal margin and is nontender. Laboratory studies show normal serum levels of AST and ALT but elevated serum levels of alkaline phosphatase (420 U/L). Endoscopic retrograde cholangiopancreatography demonstrates a beaded appearance of the extrahepatic biliary tree. Which of the following diseases is a late complication of this patient's condition? (A) Adenocarcinoma of the gallbladder (B) Cholangiocarcinoma (C) Hepatic adenoma (D) Hepatic angiosarcoma (E) Hepatocellular carcinoma

The answer is B: Cholangiocarcinoma. Primary sclerosing cholangitis (PSC) is characterized by infl ammation and obliterative fi brosis of intrahepatic and extrahepatic bile ducts, with dilation of preserved segments. Approximately 70% of patients with PSC have longstanding ulcerative colitis, although the prevalence of PSC in such patients is only 4%. PSC tends to occur in the third through fi fth decades of life, with a signifi - cant male predominance (2:1). The clinicopathologic fi ndings are complemented by a characteristic radiographic appearance of a beaded biliary tree, representing sporadic strictures. Cholangiocarcinoma is a late complication of PSC. The other choices are not complications of PSC. Diagnosis: Primary sclerosing cholangitis

A 3-year-old boy is rushed to the emergency room in acute distress. The child has vague chest pain and diffi culty swallowing. He refuses to drink water. Physical examination shows drooling and salivation. Vital signs are normal. The mother states that she saw the boy ingesting a liquid used to clear drains. If this chemical was a strong acid, which of the following histopathologic fi ndings would be expected in the esophagus of this child? (A) Apoptosis (B) Coagulative necrosis (C) Fat necrosis (D) Hyaline sclerosis (E) Liquefactive necrosis

The answer is B: Coagulative necrosis. Chemical injury to the esophagus usually refl ects accidental poisoning in children, attempted suicide in adults, or contact with medication. Ingestion of strong acids produces an immediate coagulative necrosis in the esophagus, which results in a protective eschar that limits injury and further chemical penetration. By contrast, ingestion of strong alkaline solutions is accompanied by liquefactive necrosis (choice E), with infl ammation and saponifi cation of membrane lipids. Alkaline solutions are particularly insidious because they are generally odorless and tasteless and, therefore, easily swallowed before protective refl exes come into play. Diagnosis: Chemical esophagitis

A 74-year-old woman presents with 3 weeks of left lower quadrant abdominal pain, changes in bowel habits, and intermittent fever. Her temperature is 38°C (101°F), respirations are 19 per minute, and blood pressure is 130/80 mm Hg. Physical examination shows left lower quadrant tenderness. A CBC reveals neutrophilia. An abdominal-pelvic ultrasound examination is normal. Which of the following is the most likely diagnosis? (A) Appendicitis (B) Diverticulitis (C) Ovarian carcinoma (D) Renal colic (E) Uterine leiomyoma

The answer is B: Diverticulitis. Diverticular disease refers to two entities: a condition termed diverticulosis and an infl ammatory complication called diverticulitis. Diverticulosis is generally asymptomatic. Diverticulitis results from the irritation caused by retained fecal material that obstructs the lumen of a diverticulum. Clinically, the most common symptoms of diverticulitis usually follow microscopic or gross perforation of the diverticulum. Diverticula are most common in the sigmoid colon, which is affected in 95% of cases. Peritonitis and sepsis are serious complications. Appendicitis (choice A) usually presents with right lower quadrant pain. None of the other choices presents with gastrointestinal symptoms and fever. Diagnosis: Diverticulitis

A 45-year-old woman presents with general discomfort and increasing tightness in the skin of her face. She reports intermittent pain in the tips of her fi ngers when exposed to the cold. Physical examination shows "stone facies" and edema of the fi ngers and hands. Serologic tests for antinuclear and anti-Scl-70 antibodies are both positive. Which of the following gastrointestinal manifestations is expected in this patient? (A) Adenocarcinoma of the esophagus (B) Dysphagia (C) Esophageal rupture (D) Esophageal varices (E) Squamous cell carcinoma of the esophagus

The answer is B: Dysphagia. This patient exhibits signs of scleroderma (progressive systemic sclerosis), which is characterized by vasculopathy and excessive collage deposition in the skin and internal organs. Patients often suffer from intermittent episodes of ischemia of the fi ngers, marked by pallor, paresthesias, and pain (Raynaud phenomenon). Anti-Scl-70 antibodies to nuclear topoisomerase are virtually specifi c for this autoimmune disease. Scleroderma can involve any portion of the gastrointestinal tract, although esophageal dysfunction is the most common and troublesome complication. The disease affects principally the lower esophageal sphincter, which may become so impaired that the lower esophagus and upper stomach are no longer distinct functional entities. In some affected patients, there may be a lack of peristalsis in the entire esophagus. The other choices are not associated with scleroderma. Diagnosis: Scleroderma

A 42-year-old man presents with long-standing abdominal pain after meals, which is relieved by over-the-counter antacids. The patient has lost 9 kg (20 lb) in the past year. Physical examination reveals peripheral edema and ascites. Laboratory studies show decreased serum albumin but normal serum levels of transaminases and gastrin. Gross and microscopic examination of this patient's stomach would most likely show which of the following pathologic changes? (A) Atrophic gastritis (B) Enlarged rugal folds (C) Intestinal metaplasia (D) Multiple hemorrhagic ulcers (E) Proliferation of neuroendocrine cells

The answer is B: Enlarged rugal folds. Ménétrier disease (hyperplastic hypersecretory gastropathy) is an uncommon disorder of the stomach characterized by enlarged rugae. It is often accompanied by a severe loss of plasma proteins (including albumin) from the altered gastric mucosa. The disease occurs in two forms: a childhood form due to cytomegalovirus infection and an adult form attributed to overexpression of TGF-α. The folds of the greater curvature in the fundus and body of the stomach (occasionally in the antrum) are increased in height and thickness, forming a convoluted brain-like surface. The other choices do not feature protein-losing enteropathy. Diagnosis: Ménétrier disease

A 56-year-old woman comes to the physician after noticing multiple lumps in her neck. Physical examination shows enlarged and nontender supraclavicular lymph nodes. Upper endoscopy discloses thickening of the gastric mucosa, without an obvious tumor. The results of gastric biopsy are shown in the image. Which of the following is the most likely diagnosis? (A) Adenocarcinoma (B) Gastric lymphoma (C) Leiomyosarcoma (D) Linitis plastica (E) Ménétrier disease

The answer is B: Gastric lymphoma. Gastric lymphoma is the most common form of extranodal lymphoma, accounting for 20% of all such tumors. Gastric lymphoma has a considerably better prognosis than gastric carcinoma (45% 5-year survival). The clinical symptoms of gastric lymphoma are nonspecifi c and indistinguishable from those of gastric carcinoma (e.g., weight loss, dyspepsia, and abdominal pain). Most gastric lymphomas are low-grade B-cell neoplasms of the MALToma (mucosa-associated lymphoid tissue) type, which arise in the setting of chronic Helicobacter pylori gastritis with lymphoid hyperplasia. Some of these lymphomas regress after eradication of the infection. The other choices do not demonstrate the lymphoid lesion depicted here. Diagnosis: Gastric lymphoma, MALToma

A 55-year-old man undergoes routine colonoscopy. A small, raised, mucosal nodule measuring 0.4 cm in diameter is identifi ed in the rectum and resected. The surgical specimen is shown in the image. Microscopic examination reveals goblet cells and absorptive cells with exaggerated crypt architecture, but no signs of nuclear atypia. Which of the following is the most likely diagnosis? (A) Adenocarcinoma (B) Hyperplastic polyp (C) Infl ammatory polyp (D) Peutz-Jeghers polyp (E) Villous adenoma

The answer is B: Hyperplastic polyp. Hyperplastic polyps are small, sessile mucosal excrescences that display exaggerated crypt architecture. They are the most common polypoid lesions of the colon and are particularly frequent in the rectum. The crypts of hyperplastic polyps are elongated and may exhibit cystic dilations. The epithelium is composed of goblet cells and absorptive cells, without any dysplasia. There are no dysplastic features indicative of adenocarcinoma (choice A). Villous adenomas (choice E) are considerably larger and exhibit prominent thin, tall, fi ngerlike processes. Peutz- Jeghers polyps (choice D) are hamartomatous. Diagnosis: Gastrointestinal polyp, hyperplastic polyp

A 45-year-old woman presents with a 6-month history of fatigue and swelling in her neck. Physical examination shows a goiter. A CBC discloses megaloblastic anemia and a normal reticulocyte count. Additionally, there is an elevated serum level of TSH and antithyroid antibodies. Needle aspiration of the left lobe of the thyroid reveals benign follicular cells and numerous lymphocytes. Anemia in this patient is most likely caused by antibodies directed to which of the following targets? (A) Chief cells (B) Intrinsic factor (C) Paneth cells (D) TSH receptor (E) Vitamin D

The answer is B: Intrinsic factor. This patient has chronic lymphocytic thyroiditis (Hashimoto thyroiditis) and pernicious anemia. Pernicious anemia is a megaloblastic anemia that is caused by malabsorption of vitamin B12 due to a defi - ciency of the intrinsic factor. In many cases, pernicious anemia is associated with other autoimmune diseases (e.g., Hashimoto thyroiditis, Graves disease, Addison disease, or diabetes mellitus type 1). Circulating antibodies to parietal cells, some of which are cytotoxic in the presence of complement, occur in 90% of patients with pernicious anemia. Two thirds of patients display an antibody to the intrinsic factor that prevents its combination with vitamin B12, thereby preventing formation of the complex that is later absorbed in the ileum. Half of all patients with pernicious anemia have circulating antibodies to thyroid tissue. Diagnosis: Pernicious anemia, Hashimoto thyroiditis

A 2-year-old boy is brought to the emergency room with a 48-hour history of nausea, vomiting, and abdominal discomfort. Physical examination reveals right lower quadrant guarding. Ultrasound examination of the abdomen reveals a 2-cm mass in the right iliac fossa. A segment of the small intestine is removed (shown in the image). Which of the following best describes this pathologic fi nding? (A) Intestinal infarct (B) Intussusception (C) Meckel diverticulum (D) Peutz-Jeghers polyps (E) Volvulus

The answer is B: Intussesception. Mechanical obstruction to the passage of intestinal contents can be caused by (1) a luminal mass, (2) an intrinsic lesion of the bowel wall, or (3) extrinsic compression. Obstruction in this case was caused by intussusception, in which a segment of bowel (intussusceptum) protruded distally into a surrounding outer portion (intussuscipiens). This condition is usually a disorder of infants or young children, in whom it occurs without a known cause. In adults, the leading point of an intussusception is usually a lesion in the bowel wall, such as Meckel diverticulum or a tumor. Once the leading point is entrapped in the intussuscipiens, peristalsis drives the intussusceptum forward. In addition to acute intestinal obstruction, intussusception compresses the blood supply to the intussusceptum, which may become infarcted. If the obstruction is not relieved spontaneously, treatment requires surgery. None of the other choices display "telescoping" of the small intestine. Meckle diverticulum (choice C) is an outpouching of the gut caused by perisitence of the embryonic vitelline duct. It is the most common congenital anomaly of the small intestine and is usually asymptomatic. Peutz-Jeghers polyps (choice D) are hamartomas of the small intestine. Volvulus (choice E) is an example of intestinal obstruction and acute abdomen, in which a segment of the gut twists on its mesentery, kinking the bowel and usually interrupting its blood supply. Diagnosis: Intussusception

A 1-year-old girl is brought to the emergency room by her parents who report that she had a fever and diarrhea for 3 days. The child's temperature is 38°C (101°F). The CBC shows a normal WBC count and increased hematocrit. Which of the following microorganisms is the most likely cause of diarrhea in this young child? (A) Cytomegalovirus (B) Rotavirus (C) Salmonella typhi (D) Shigella dysenteriae (E) Yersinia jejuni

The answer is B: Rotavirus. Rotavirus is the most common cause of infantile diarrhea and can be demonstrated in duodenal biopsy specimens in half the cases of acute diarrhea in hospitalized children under the age of 2 years. Choices C, D, and E can cause diarrhea, but are uncommon in developed countries. Diagnosis: Viral diarrhea

An 18-year-old man presents with a 2-week history of yellow skin and sclerae but is otherwise asymptomatic. He recalls a similar episode 2 years previously. His brother also has recurrent jaundice. The serum bilirubin is 5.2 mg/dL, mostly in the unconjugated form. Serum AST and ALT levels are normal, as is the urinalysis. Two weeks later, the jaundice resolves spontaneously. What is the most likely diagnosis? (A) α1-Antitrypsin defi ciency (B) Dubin-Johnson syndrome (C) Gilbert syndrome (D) Hereditary hemochromatosis (E) Wilson disease

The answer is C: Gilbert syndrome. Gilbert syndrome is an inherited, mild, recurrent, unconjugated hyperbilirubinemia (<6mg/dL) that is caused by impaired clearance of bilirubin in the absence of any detectable liver disease. The syndrome tends to run in families, and both autosomal dominant and recessive patterns have been observed. Aside from jaundice, patients are asymptomatic. Factors that increase serum bilirubin concentration in normal persons, such as fasting or illness, produce an exaggerated increase in serum bilirubin levels in persons with Gilbert syndrome. Although Dubin-Johnson syndrome (choice B) is familial, it presents with conjugated hyperbilirubinemia. The other choices do not present with episodic jaundice. Diagnosis: Gilbert syndrome

A 30-year-old man presents with a 9-month history of fatigue and recurrent fever. He also complains of yellow skin and sclerae, abdominal tenderness, and dark urine. Physical examination reveals jaundice and mild hepatomegaly. Laboratory studies demonstrate elevated serum bilirubin (3.1 mg/ dL), decreased serum albumin (2.5 g/dL), and prolonged prothrombin time (17 seconds). Serologic tests reveal antibodies to hepatitis B core antigen (IgG anti-HBcAg). The serum is positive for HBsAg and HbeAg. A liver biopsy is shown in the image. What is the most likely diagnosis? (A) Acute hepatitis B (B) Alcoholic hepatitis (C) Chronic hepatitis B (D) Delta virus infection (E) Subacute hepatic necrosis secondary to hepatitis B infection

The answer is C: Chronic hepatitis B. Chronic hepatitis B refers to infection with hepatitis B virus (HBV) that is associated with necrosis and infl ammation in the liver for more than 6 months. HBV is a DNA virus that is transmitted through blood transfusion, sexual contact, or shared needles. Most patients recover completely from acute infection, but some 10% develop chronic infection. Of the latter, 10% to 30% develop chronic hepatitis and cirrhosis. The biopsy in this case shows hepatocellular nodules and chronically-infl amed fi brous septa (see photomicrograph). Surface antigen (HBsAg) is present in the serum of patients with chronic hepatitis B, and the presence of HbeAg is often associated with progression of the disease. Choices A, B, and E do not demonstrate cirrhosis as depicted and do not show the serologic characteristics of HBV infection. Diagnosis: Hepatitis B, chronic

A 66-year-old man presents with a 2-week history of abdominal bloating, weight loss, and pain in the right upper quadrant. The patient had a serious motor vehicle accident 16 years ago, in which he required transfusion of 10 U of whole blood. On physical examination, he exhibits massive distension of the abdomen. The liver is hard on palpation and occupies the entire right side of the abdomen. Laboratory studies show a low serum albumin (2.2 g/dL) and a markedly elevated serum alpha-fetoprotein. An abdominal ultrasound examination reveals ascites. The patient expires 6 months later. The liver at autopsy is shown in the image. Which of the following is the most common cause of this disease worldwide? (A) Alcoholic hepatitis (B) Autoimmune hepatitis (C) Chronic hepatitis B (D) Chronic hepatitis C (E) Hepatitis E

The answer is C: Chronic hepatitis B. Patients with persistent hepatitis B virus (HBV) infection have a 200-fold increased risk of developing primary hepatocellular carcinoma (HCC), the diagnosis in this case. More than 85% of cases of HCC occur in countries with a high prevalence of chronic infection with HBV. One fourth of patients with chronic hepatitis B ultimately develop HCC. Chronic hepatitis C (choice D) is associated with most cases of HCC in Europe and North America, but chronic hepatitis B remains the major global cause of HCC. Diagnosis: Hepatitis B, chronic; hepatocellular carcinoma

A 62-year-old man is brought to the emergency room in a disoriented state. Physical examination reveals signs of poor hygiene and an odor of alcohol, as well as jaundice, splenomegaly, and ascites. The patient has a coarse fl apping tremor of the hands, palmar erythema, and diffuse spider angiomata. The abdomen displays dilated paraumbilical veins. Serum levels of ALT, AST, alkaline phosphatase, and bilirubin are all mildly elevated. Soon after admission, the patient vomits a large amount of blood. Which of the following is the most likely underlying cause of hematemesis in this patient? (A) Acute alcoholic hepatitis (B) Acute gastritis (C) Cirrhosis (D) Hepatic steatosis (E) Mallory-Weiss tear

The answer is C: Cirrhosis. Cirrhosis represents the end stage of chronic liver disease and is characterized by extensive fi brosis and the formation of regenerative nodules. Patients with cirrhosis often present with complications of portal hypertension, including ascites, splenomegaly, and bleeding esophageal varices. Esophageal varices arise from the opening of portal-systemic venous collaterals. Engorged collaterals in the submucosa of the lower esophagus and upper stomach which dilate and protrude into the lumen, are susceptible to bleeding. The prognosis of patients with bleeding esophageal varices is poor, with a 40% mortality rate. Mallory-Weiss tear (choice E) is a possible cause of hematemesis, but is only seen in patients with protracted vomiting. The other choices do not present with portal hypertension or systemic signs of endstage liver disease. Diagnosis: Alcoholic cirrhosis, bleeding esophageal varices

A 21-year-old man is brought to the emergency room with symptoms of acute intestinal obstruction. His temperature is 38°C (101°F), respirations are 25 per minute, and blood pressure is 120/80 mm Hg. Physical examination reveals a mass in the right lower abdominal quadrant. The patient subsequently undergoes surgery, and a segmental lesion involving the terminal ileum is resected (shown in the image). Which of the following is the most likely diagnosis? (A) Adenocarcinoma (B) Carcinoid tumor (C) Crohn disease (D) Pseudomembranous colitis (E) Ulcerative colitis

The answer is C: Crohn disease. Crohn disease is a transmural, chronic infl ammatory disease that may affect any part of the digestive tract but occurs principally in the distal small intestine and occasionally the right colon. It has variously been referred to as terminal ileitis and regional ileitis when it involves the ileum and granulomatous colitis when it principally affects the colon. Skip lesions are common. The affected mucosa has a characteristic "cobblestone" appearance (shown in the image) due to the presence of linear ulcerations and edema, and infl ammation of the intervening tissue. The other choices do not show the characteristic cobblestone morphology that is seen in this case. Diagnosis: Crohn disease

A 24-year-old man is brought to the emergency room with symptoms of acute intestinal obstruction. His temperature is 38°C (101°F), respirations are 25 per minute, and blood pressure is 120/80 mm Hg. Physical examination reveals a mass in the right lower abdominal quadrant. At laparoscopy, there are numerous small bowel strictures and a fi stula extending into a loop of small bowel. Which of the following is the most likely diagnosis? (A) Adenocarcinoma (B) Carcinoid tumor (C) Crohn disease (D) Pseudomembranous colitis (E) Ulcerative colitis

The answer is C: Crohn disease. Crohn disease is a transmural, chronic infl ammatory disease that may affect any part of the digestive tract. Intestinal obstruction and fi stulas are the most common intestinal complications of Crohn disease. Occasionally, free perforation of the bowel occurs. The risk of small bowel cancer is increased at least threefold in patients with Crohn disease. Pseudomembranous colitis (choice D) and ulcerative colitis (choice E) are not associated with fi stula formation. Adenocarcinoma (choice A) rarely, if ever, arises in the terminal ileum. Diagnosis: Crohn disease

A 25-year-old heroin addict presents in a disoriented state with a 5-day history of fatigue, malaise, and dark-colored urine. Physical examination reveals jaundice and multiple petechial hemorrhages on the upper extremities. Laboratory studies show serum bilirubin of 15.6 mg/dL, mostly in the conjugated form, 10-fold elevations of serum AST and ALT, high levels of blood ammonia, and increased prothrombin time (15 seconds). The patient's condition deteriorates and he develops stage 4 hepatic encephalopathy. A liver biopsy is shown in the image. Which of the following viruses is most likely responsible for the clinical and pathologic fi ndings in this patient? (A) Cytomegalovirus (B) Hepatitis A virus (C) Hepatitis B virus (D) Hepatitis C virus (E) Hepatitis E virus

The answer is C: Hepatitis B virus. Massive hepatic necrosis often leads to fulminant hepatic failure. A common cause of massive hepatic necrosis is hepatitis B virus. The liver appears shrunken, the capsule is wrinkled, and the parenchymal tissue is soft and fl abby. On microscopic examination, the necrotic liver lobules are hemorrhagic, and the reticulin framework has collapsed. Hepatitis A virus, C virus, and E virus (choices B, D, and E) rarely present with massive hepatic necrosis. Diagnosis: Fulminant hepatitis B infection, hepatic failure

A 34-year-old man with AIDS presents with a 3-month history of constipation and lower abdominal pain. The patient has a history of chronic diarrhea and persistent cough. Recently, he noticed blood in his stool. Laboratory studies reveal mild irondefi ciency anemia. Stool specimens are positive for occult blood. A CBC shows a CD4 count of less than 50/μL. Sigmoidoscopy discloses a mass in the rectosigmoid region. In addition to B-cell lymphoma, this patient is at increased risk of developing which of the following tumors of the gastrointestinal (GI) tract? (A) Carcinoid tumor (B) Colonic adenocarcinoma (C) Kaposi sarcoma (D) Leiomyosarcoma (E) Melanoma

The answer is C: Kaposi sarcoma. Kaposi sarcoma of the GI tract is found almost exclusively in patients with AIDS. One third to one half of AIDS patients, with cutaneous Kaposi sarcoma, exhibit involvement of the GI tract. In most patients, intestinal Kaposi sarcoma does not lead to symptoms, although GI bleeding, obstruction, and malabsorption have been reported. The other choices are not specifi cally associated with AIDS. Diagnosis: AIDS, Kaposi sarcoma

Physical examination of a newborn female after delivery reveals an imperforate anus. Meconium is visible behind the thin, cutaneous membrane. The classifi cation of this anorectal malformation is based on the relationship of the terminal bowel to which of the following anatomic structures? (A) Ganglia in the wall of the rectum (B) Inferior mesenteric artery (C) Levator ani muscle (D) Muscularis mucosae of rectum (E) Urachus

The answer is C: Levator ani muscle. Anorectal malformations are among the most common anomalies and vary from minor narrowing to serious and complex effects. The classifi cation of these anomalies is based on the relation of the terminal bowel to the levator ani muscle. The other choices are not associated with anorectal malformations. Diagnosis: Anorectal malformation

A previously healthy, 24-year-old woman presents with a 1-week history of intermittent fever, lethargy, and yellow skin and sclerae. Physical examination shows jaundice. Laboratory studies reveal decreased serum albumin (2.2 g/dL), extremely high levels of AST and ALT (1,200 and 1,800 U/L, respectively), and elevated alkaline phosphatase (300 U/L). Her ceruloplasmin level is normal. She is admitted to the hospital. Her condition progressively deteriorates, and she develops hepatic encephalopathy and hepatorenal syndrome. Which of the following is the most likely diagnosis? (A) Extrahepatic biliary obstruction (B) Hereditary hemochromatosis (C) Massive hepatic necrosis (D) Primary biliary cirrhosis (E) Sclerosing cholangitis

The answer is C: Massive hepatic necrosis. Massive hepatic necrosis is the most feared variant of acute hepatitis. The laboratory fi ndings seen in this patient (markedly elevated ALT and AST) are characteristic for this condition. Grossly, the liver loses about one third of its normal weight, and Glisson capsule is wrinkled and mottled. Microscopic examination reveals massive death of the hepatocytes, leaving a collapsed collagenous framework. Although the other choices can lead to hepatic failure, patients are typically symptomatic prior to hepatic decompensation. Diagnosis: Massive hepatic necros

A 58-year-old woman suffers a massive stroke and expires. The stomach at autopsy is shown in the image. Prior to her death, this patient would most likely have exhibited which of the following? (A) Dysphagia (B) Hypersecretion of gastric acid (C) Melena (D) Steatorrhea (E) Variceal bleeding

The answer is C: Melena. Peptic ulcers of the stomach and duodenum are estimated to affl ict 10% of the population of Western industrialized countries at some time during their lives. Peptic ulcers appear as punched out, rounded ulcers. Erosion through arteries causes bleeding and iron-defi ciency anemia. Melena refers to black, tarry stools composed largely of blood from the upper digestive tract that has been processed by the action of gastric juices. Melena is commonly seen in patients who suffer from chronic peptic ulcer disease. Unlike duodenal ulcers, most patients with gastric ulcers secrete normal or decreased amounts of gastric acid (see choice B). Diagnosis: Peptic ulcer disease, gastric ulcer

A 58-year-old man with longstanding alcoholic cirrhosis presents with abdominal pain, fever, and an episode of hematemesis. Physical examination reveals jaundice and a markedly distended abdomen. The patient is disoriented and has a coarse fl apping tremor of the hands. Laboratory studies reveal modestly elevated serum levels of AST and ALT (96 and 92 U/L, respectively) and a high serum level of alkaline phosphatase (320 U/L). Prothrombin time is prolonged (20 seconds). The WBC count is 18,000/μL. Shortly after admission, the patient develops coma, adult respiratory distress syndrome, and renal failure (oliguria and elevated serum levels of BUN and creatinine), leading to death within 3 days. Histologic examination of the patient's kidney at autopsy would most likely show which of the following? (A) Interstitial nephritis (B) Membranous nephropathy (C) No histologic changes (D) Proliferative glomerulonephritis (E) Pyelonephritis

The answer is C: No histologic changes. Hepatorenal syndrome usually occurs in the setting of cirrhosis and heralds a poor prognosis. The disorder is characterized by features of renal hypoperfusion, including oliguria, azotemia, and increased levels of serum creatinine. Microscopically, the kidney appears normal. Renal failure is caused by vasoconstriction and hypoperfusion of the kidneys, a combination mediated by various hormones and vasoactive substances, some of which may not be cleared by the cirrhotic liver. Similarly, a kidney from a patient in hepatorenal failure may be successfully transplanted into another person and assume normal functioning. The other choices are associated with direct injury to the renal parenchyma and exhibit characteristic histologic fi ndings. Diagnosis: Hepatorenal syndrome

A 5-year-old girl is brought to the physician after her parents noticed red blood in her stool. Physical examination reveals mucocutaneous pigmentation. Small bowel radiography discloses multiple, small- to medium-sized polyps that are diagnosed pathologically as hamartomas. Which of the following is the most likely diagnosis? (A) Congenital teratoma (B) Hyperplastic polyp (C) Peutz-Jeghers polyp (D) Tubular adenoma (E) Villous adenoma

The answer is C: Peutz-Jeghers polyp. Peutz-Jeghers syndrome is an autosomal dominant, hereditary disorder characterized by intestinal hamartomatous polyps and mucocutaneous melanin pigmentation, which is particularly evident on the face, buccal mucosa, hands, feet, and perianal and genital regions. The polyps seen in Peutz-Jeghers syndrome are hamartomatous, characterized by a branching network of smooth muscle fi bers continuous with the muscularis mucosa that support the glandular epithelium of the polyp. Congenital teratoma (choice A) does not involve the intestine. The other choices are principally colonic polyps that derive from the luminal epithelium. Diagnosis: Gastrointestinal polyp, Peutz-Jeghers polyp

A 50-year-old obese man (BMI = 32 kg/m2) comes to the physician complaining of indigestion after meals, bloating, and heartburn. Vital signs are normal. A CT scan of the abdomen reveals a hiatal hernia of the esophagus. Endoscopic biopsy shows thickening of the basal layer of the squamous epithelium, upward extension of the papillae of the lamina propria, and an increased number of neutrophils and lymphocytes. Which of the following is the most likely diagnosis? (A) Esophageal varices (B) Mallory-Weiss syndrome (C) Refl ux esophagitis (D) Schatzki mucosal ring (E) Squamous cell carcinoma

The answer is C: Refl ux esophagitis. Esophagitis may be caused by infections, refl ux of gastric juice, or exogenous irritants. Of these, the most common type is refl ux esophagitis, which is often found in conjunction with a sliding hiatal hernia but may also arise through an incompetent lower esophageal sphincter without any demonstrable anatomical lesion. Chronic exposure to stomach juice causes reactive thickening of the squamous epithelium (leukoplakia) and the underlying stroma. Areas affected by gastric refl ux are susceptible to mucosal erosions and ulcers which appear as linear vertical streaks. Neutrophils and lymphocytes accumulate in the mucosa. The other choices are not typical complications of hiatal hernia. Diagnosis: Refl ux esophagitis, hiatal hernia

A 65-year-old woman presents with a 3-month history of diarrhea and abdominal pain. She has lost 9 kg (20 lb) in the past 6 months. The patient had two benign colonic polyps removed 3 years ago. Laboratory studies reveal mild iron-defi ciency anemia, and stool specimens are positive for occult blood. Sigmoidoscopy demonstrates an ulcerated mass, and a biopsy shows malignant glands. A segment of the colon is resected, and the surgical specimen is shown in the image. Based on current models of colonic carcinogenesis, which of the following genes was most likely mutated in the transition from benign adenoma to carcinoma in this patient? (A) BRCA1 (B) C-myc (C) p53 (D) Ras (E) VHL

The answer is C: p53. In most cases of colorectal carcinoma, it has been estimated that a minimum of eight to ten mutational events must accumulate before the development of invasive cancer. This process is initiated in morphologically normal mucosa, proceeds through an adenomatous precursor, and terminates as invasive adenocarcinoma. The APC gene is considered to play an important role in the early development of most colorectal neoplasms, whereas mutations in the p53 tumor suppressor gene are thought to participate in the late transition from adenoma to carcinoma. BRCA1 (choice A) has been implicated in the pathogenesis of breast and ovarian cancers. VHL (choice E) has been incriminated in the pathogenesis of clear cell renal cell carcinoma. Diagnosis: Adenocarcinoma of colon

A 20-year-old woman presents with a 2-year history of diffi culty swallowing and increasing fatigue. A CBC shows iron-defi ciency anemia. Upper endoscopy reveals an annular narrowing in the upper third of the esophagus. A mucosal biopsy shows no evidence of infl ammation or neoplasia. Which of the following is the most likely diagnosis? (A) Achalasia (B) Barrett esophagus (C) Diverticulum (D) Esophageal web (E) Schatzki ring

The answer is D: Esophageal web. Esophageal rings and webs cause dysphagia. Webs are thin mucosal membranes that project into the lumen of the esophagus. Rings are thicker than webs and contain smooth muscle. The clinical symptoms of esophageal webs and rings include dysphagia, esophageal substernal pain, and aspiration or regurgitation of foods and liquids. Plummer-Vinson syndrome is characterized by a cervical esophageal web, mucosal lesions of the mouth and pharynx, and iron-defi ciency anemia. Carcinoma of the oropharynx and upper esophagus are complications of Plummer-Vinson syndrome. The other choices are not associated with anemia. Schatzki ring (choice E) occurs near the gastroesophageal junction. Diagnosis: Esophageal web

For the patient described in Question 26, which of the following is a common complication? (A) Bile peritonitis (B) Chronic passive congestion of the liver (C) Confl uent hepatic necrosis (D) Extrahepatic biliary obstruction (E) Primary hepatocellular carcinoma

The answer is D: Extrahepatic biliary obstruction. Most complications associated with cholelithiasis are related to obstruction of the biliary tree. Passage of stones into the cystic duct often causes severe biliary colic. Lodgement of stones in the common bile duct leads to obstructive jaundice, cholangitis, and acute pancreatitis. Patients with cholelithiasis have a 25-fold increased risk of acute pancreatitis compared with the general population. Additional complihistologic cations are rare and include empyema of the gallbladder, perforation, fi stula formation, bile peritonitis, and gallstone ileus. In most cases, gallstones are associated with chronic cholecystitis. Choices B, C, and E are not associated with gallstones. Diagnosis: Cholelithiasis, obstructive jaundice

In addition to anorectal malformation, the infant described in Question 58 is most likely to have which of the following birth defects? (A) Cleft lip/cleft palate (B) Congenital pyloric stenosis (C) Esophageal atresia (D) Gastrointestinal fi stula (E) Persistent urachus

The answer is D: Gastrointestinal fi stula. Anorectal malformations result from arrested development of the caudal region of the gut in the fi rst 6 months of fetal life. The cause is unknown. Fistulas between the malformation and the bladder, urethra, vagina, or skin may occur in all types of anorectal anomalies. The other choices are not associated with anorectal malformations. Diagnosis: Anorectal malformation

A 54-year-old man with a long history of indigestion after meals and "heartburn" presents with upper abdominal pain. He was treated with proton-pump inhibitors for gastroesophageal refl ux 3 years previously. An endoscopic biopsy of the lower esophagus is shown in the image. Which of the following best describes these pathologic fi ndings? (A) Candida esophagitis (B) Esophageal diverticulum (C) Esophageal varices (D) Glandular metaplasia (E) Schatzki ring

The answer is D: Glandular metaplasia The biopsy shows Barrett esophagus, which is defi ned as a replacement of the esophageal squamous epithelium by columnar epithelium as a result of chronic gastroesophageal refl ux. This disorder occurs typically in the lower third of the esophagus. The lesion is characterized histologically by distinctive intestine-like epithelium composed of goblet cells and surface cells similar to those of incompletely intestinalized gastric mucosa. Complete intestinal metaplasia, with Paneth cells and absorptive cells, may occur. Barrett esophagus is more resistant to peptic juices than normal squamous epithelium and appears to be an adaptive mechanism that serves to limit the harmful effects of gastroesophageal refl ux. None of the other choices lead to metaplastic changes. Diagnosis: Barrett esophagus, refl ux esophagitis

A 45-year-old man presents with long-standing heartburn and dyspepsia. An X-ray fi lm of the chest shows a retrocardiac, gas-fi lled structure. This patient most likely has which of the following conditions? (A) Boerhaave syndrome (B) Esophageal varices (C) Esophageal webs (D) Hiatal hernia (E) Mallory-Weiss syndrome

The answer is D: Hiatal hernia. Hiatal hernia is a protrusion of the stomach through an enlarged esophageal hiatus in the diaphragm. Two basic types of hiatal hernia are observed. In sliding hiatal hernias, an enlargement of the hiatus and laxity of the circumferential connective tissue allows a cap of gastric mucosa to move upward above the diaphragm. Paraesophageal hiatal hernias are characterized by herniation of a portion of the gastric fundus alongside the esophagus through a defect in the diaphragmatic connective tissue membrane that defi nes the esophageal hiatus. Symptoms of hiatal hernia, particularly heartburn and regurgitation, are attributed to the refl ux of gastric contents, which is primarily related to incompetence of the lower esophageal sphincter. Classically, the symptoms are exacerbated when the affected person is recumbent. Large herniations carry a risk of gastric volvulus or intrathoracic gastric dilation. Boerhaave syndrome (choice A) represents rupture of the esophagus as a result of vomiting. Mallory-Weiss syndrome (choice E) refers to mucosal laceration of the upper stomach and lower esophagus in the setting of severe retching. Diagnosis: Paraesophagic hiatal hernia

Which of the following cellular/biochemical mechanisms best explains the pathogenesis of malabsorption in the patient described in Question 54? (A) Bile salt inactivation (B) Blind loop syndrome (C) Extrahepatic cholestasis (D) Impaired mucosal function (E) Obstruction of the common bile duct

The answer is D: Impaired mucosal function. Normal intestinal absorption is characterized by a luminal phase and an intestinal phase. The intestinal phase includes those processes that occur in epithelial cells and transport channels. Injury to the mucosa in patients with Whipple disease (secondary to infl ammation) results in impaired transport of nutrients through the intestinal wall. Histologic examination of the small intestine reveals fl at, thickened villi and extensive infi ltration of the lamina propria with foamy macrophages. The other choices pertain to luminal phase processes that are unaffected in patients with Whipple disease. Diagnosis: Whipple disease

For the patient described in Question 21, a liver biopsy would most likely show which of the following pathologic fi ndings? (A) Central hyaline sclerosis (B) Cholangiocarcinoma (C) Hemosiderosis (D) Intrahepatic bile duct damage (E) Macrovesicular steatosis

The answer is D: Intrahepatic bile duct damage. Primary biliary cirrhosis (nonsuppurative destructive cholangitis) is caused by chronic destruction of intrahepatic bile ducts in the portal tracts. Primary biliary cirrhosis evolves through ductal lesions, scarring, and eventually cirrhosis. Early PBC features chronic destructive cholangitis affecting intrahepatic small and medium-sized bile ducts. The bile ducts are surrounded primarily by lymphocytes (CD8+ T cells), but plasma cells and macrophages are also seen. In some portal tracts, lymphoid follicles are conspicuous. Discrete epithelioid granulomas often occur in the portal tracts and may impinge on the bile ducts. As a result of the destructive chronic infl ammatory process, small bile ducts virtually disappear, and scarring of medium-sized bile ducts is common. Proliferation of bile ductules within portal tracts is common and may be fl orid. Collagenous septae extend from the portal tracts into the lobular parenchyma and encircle some lobules. Cholestasis, when present, may be severe and is located at the periphery of the portal tracts. The end-stage of PBC is cirrhosis, characterized by a dark green bile-stained liver that exhibits fi ne nodularity. The other choices do not feature destruction of intrahepatic bile ducts. Diagnosis: Primary biliary cirrhosis

A 58-year-old woman presents with a 2-month history of abdominal discomfort and dark stools. Physical examination shows pallor but no evidence of jaundice. Laboratory studies disclose a microcytic, hypochromic anemia, with a hemoglobin level of 6.7 g/dL. A barium swallow radiograph reveals a "leather bottle" appearance of the stomach. Microscopic examination shows diffusely infi ltrating malignant cells, many of which are "signet ring" cells, in the stomach wall. Which of the following is the most likely diagnosis? (A) Fungating adenocarcinoma (B) Gastric leiomyosarcoma (C) Gastric lymphoma (D) Linitis plastica (E) Ménétrier disease

The answer is D: Linitis plastica. Diffuse adenocarcinoma constitutes10% of all stomach cancers. No true tumor mass is seen macroscopically. Instead, the wall of the stomach is conspicuously thickened and fi rm, accounting for the radiologic "leather bottle" appearance. When the entire stomach is involved, the term linitis plastica is applied. The invading tumor cells induce extensive fi brosis in the submucosa and muscularis of the stomach wall. Gastric carcinomas typically metastasize to regional lymph nodes and the liver. Signet ring cells are so named because intracellular mucin displaces the nuclei to the periphery of the tumor cells. Gastric carcinomas and linitis plastica, in particular, have a poor prognosis. The other choices do not show the characteristic morphologic appearance of linitis plastica and generally do not exhibit signet ring cells. Diagnosis: Gastric adenocarcinoma

A 25-year-old woman complains of sudden onset of acute abdominal pain. Physical examination shows abdominal distention. Her temperature is 37°C (98.6°F), respirations 22 per minute, heart rate 110 per minute, and blood pressure 70/50 mm Hg. A tap of the abdomen returns blood. A CT scan reveals a solitary 20-cm mass of the liver. A surgically resected portion of the liver is shown in the image. This patient's tumor was most likely associated with chronic exposure to which of the following? (A) Carbon tetrachloride (B) Halothane (C) L-thyroxine (D) Oral contraceptives (E) Vinyl chloride

The answer is D: Oral contraceptives. Hepatic adenoma usually occurs as a solitary, sharply demarcated mass up to 40 cm in diameter and 3 kg in weight. On gross examination, the tumor is encapsulated and paler than the surrounding liver parenchyma. Hepatic adenoma is a complication of oral contraceptive use in women. In about 30% of patients with hepatic adenomas, the tumor tends to bleed into the peritoneal cavity, inducing hypovolemic shock that requires emergency treatment. The other choices do not induce hepatic tumors. Diagnosis: Hepatic adenoma

The patient described in Question 8 is most likely to develop which of the following vascular infl ammatory diseases? (A) Allergic angiitis (B) Buerger disease (C) Giant cell arteritis (D) Polyarteritis nodosa (E) Wegener granulomatosis

The answer is D: Polyarteritis nodosa. Some HBV carriers manifest circulating immune complexes, which cause a variety of extrahepatic ailments, including a serum sickness-like syndrome, glomerulonephritis, cryoglobulinemia, and polyarteritis nodosa. Polyarteritis nodosa is a necrotizing arteritis of medium-sized vessels that can lead to pseudoaneurysm formation, renal thrombosis, infl ammation, and hemorrhage. The other choices are not associated with chronic hepatitis. Diagnosis: Hepatitis B, chronic; polyarteritis nodosa

A 53-year-old woman complains of acute diarrhea and severe abdominal pain. She was recently treated with broad-spectrum antibiotics for community-acquired pneumonia. A CBC shows a WBC count of 24,000/μL. The patient subsequently develops septic shock and dies. A portion of her colon is shown at autopsy. These fi ndings are typical of which of the following gastrointestinal diseases? A) Crohn disease (B) Diverticulitis (C) Ischemic colitis (D) Pseudomembranous colitis (E) Ulcerative colitis

The answer is D: Pseudomembranous colitis. Pseudomembranous colitis is a generic term for an infl ammatory disease of the colon that is characterized by exudative plaques on the mucosa. Antibiotic therapy eliminates the normal mixed fl ora of the colon and facilitates the overgrowth of Clostridium diffi cile, leading to an acute infection of the colon. The exotoxins produced by C. diffi cile cause intestinal necrosis, with superfi - cial ulcers covered by a thick fi bropurulent exudate. The other choices are not related to antibiotic therapy and are not associated with the development of these exudative plaques. Diagnosis: Pseudomembranous colitis

A 70-year-old woman with a history of ovarian cancer presents with diarrhea. She completed radiation therapy for her cancer 3 months ago. Physical examination shows cachexia, hyperactive bowel sounds, and generalized pallor. The stools are found to contain blood. A CBC shows decreased hemoglobin (7.8 g/dL) and decreased mean corpuscular volume (70 μm3). Which of the following is the most likely cause of GI bleeding in this patient? (A) Angiodysplasia (B) Hemorrhoids (C) Ischemic colitis (D) Radiation enterocolitis (E) Solitary rectal ulcer

The answer is D: Radiation enterocolitis. Radiation therapy for malignant disease of the pelvis or abdomen may be complicated by injury to the small intestine and colon. Clinically signifi cant radiation enterocolitis is most common in the rectum. The lesions produced by radiation therapy range from a reversible injury of the intestinal mucosa to chronic infl ammation, ulceration, and fi brosis of the intestine. Diagnosis: Radiation enterocolitis, ovarian cancer

A 60-year-old man presents with a 5-week history of difficulty swallowing. Physical examination is unremarkable. Upper endoscopy shows a large mass in the upper third of the esophagus. A biopsy is shown in the image. What is the appropriate histologic diagnosis for this esophageal mass? (A) Adenocarcinoma (B) Glandular metaplasia (C) Malignant melanoma (D) Squamous cell carcinoma (E) Transitional cell carcinoma

The answer is D: Squamous cell carcinoma. Most cancers of the esophagus worldwide are squamous cell carcinomas, although adenocarcinoma is now more common in the United States. Squamous cell tumors range from well-differentiated cancers with "epithelial pearls" to poorly differentiated neoplasms that lack evidence of squamous differentiation. The most common presenting complaint of patients with esophageal cancer is dysphagia, but by this time, most tumors are unresectable. Adenocarcinoma (choice A) and Barrett esophagus (choice B) are incorrect because the biopsy does not show glandular features. Primary malignant melanoma (choice C) of the esophagus is extremely rare, although melanoma metastases to the esophagus may occur. Diagnosis: Squamous cell carcinoma of the esophagus

A 23-year-old woman with a history of an eating disorder complains of vomiting, nausea, and severe abdominal pain. Physical examination shows abdominal distension and constipation. An X-ray fi lm of the abdomen reveals air-fl uid levels and a hyperlucent shadow at the epigastric area. The material obstructing the gastrointestinal tract is removed surgically and shown. Which of the following is the most likely diagnosis? (A) Diverticulum of the stomach (B) Gastric lymphoma (C) Phytobezoar (D) Trichobezoar (E) Volvulus of the stomach

The answer is D: Trichobezoar. Bezoars are foreign bodies in the stomach that are composed of food or hair that have been altered by the digestive process. The mass removed from the stomach in this patient is a hairball (trichobezoar) within a gelatinous matrix. Trichobezoar is usually seen in long-haired girls or young women who eat their own hair as a nervous habit (trichotillomania; also called "Rapunzel" syndrome). Such a trichobezoar may grow by accretion to form a complete cast of the stomach. Strands of hair may extend into the bowel as far as the transverse colon (Rapunzel syndrome). Phytobezoars (choice C) are concretions of plant material, which usually occur in persons with conditions that interfere with gastric emptying. Diagnosis: Trichobezoar, bezoar, Rapunzel syndrome

A 65-year-old woman undergoes routine colonoscopy. During the procedure, a 2-cm mass is identifi ed in the rectosigmoid region and resected. The surgical specimen is shown in the image. Microscopic examination shows irregular crypts lined by pseudostratifi ed epithelium with hyperchromatic nuclei, without dysplastic features. Which of the following is the most likely diagnosis for this patient's colonic lesion? (A) Adenocarcinoma (B) Carcinoid tumor (C) Hyperplastic polyp (D) Tubular adenoma (E) Villous adenoma

The answer is D: Tubular adenoma. Tubular adenomas constitute two thirds of benign colonic adenomas. They are typically smooth-surface lesions, usually less than 2 cm in diameter, and often have a stalk. Microscopically, tubular adenomas exhibit closely packed epithelial tubules, which may be uniform or irregular with excessive branching. Dysplasia and carcinoma often develop in tubular adenomas. As long as dysplastic foci remain confi ned to the polyp mucosa, the lesion is almost always cured by resection. Adenocarcinoma (choice A) is incorrect because the lesion does not have dysplastic features. Pseudostratifi ed epithelium is not a feature of carcinoid tumor (choice B) or hyperplastic polyp (choice C). The incorrect choices do not typically exhibit a stalk. Diagnosis: Gastrointestinal polyp, tubular adenoma

A 54-year-old man presents with a 9-month history of progressive skin pigmentation. He passes large amounts of urine and is always thirsty. His father died of liver cancer. Physical examination reveals a dark skin color and an enlarged liver. Laboratory studies show normal serum levels of corticotropin. A glucose tolerance test indicates chemical diabetes. A liver biopsy stained with Prussian blue is shown in the image. If untreated, which of the following conditions is most likely to develop in this patient? (A) Acute hepatitis (B) Addison disease (C) Cholangiocarcinoma (D) Cholelithiasis (E) Hepatocellular carcinoma

The answer is E: Development of hepatocellular carcinoma. Hereditary hemochromatosis (HH) is a common, autosomal recessive, genetic disorder that is characterized by excessive iron absorption and the toxic accumulation of iron in parenchymal cells, particularly of the liver, heart, and pancreas. In the liver, HH leads to cirrhosis and a high incidence of primary hepatocellular carcinoma. The clinical hallmark of advanced HH is the presence of other diseases such as diabetes, skin pigmentation, and cardiac failure. The Prussian blue stain binds iron and provides histologic evidence for iron overload. Addison disease (choice B) presents with skin pigmentation but refl ects autoimmune destruction of the adrenal glands. Diagnosis: Hereditary hemochromatosis

A 60-year-old woman presents with several years of abdominal pain radiating to her back and a 5-day history of yellow skin and sclerae. She has lost 15 lb during the past several months, and her stools have become lighter in color. On physical examination, the patient is cachectic and jaundiced. The liver edge descends 1 cm below the right costal margin and is nontender. Her right calf is tender and erythematous. Serum AST and ALT are at the upper limits of normal, but alkaline phosphatase is increased to 430 U/L. A CT scan shows a mass in the head of the pancreas. What is the most likely cause of jaundice in this patient? (A) Acute viral hepatitis (B) Alcoholic hepatitis (C) α1-Antitrypsin defi ciency (D) Drug-induced hepatitis (E) Extrahepatic biliary obstruction

The answer is E: Extrahepatic biliary obstruction. This patient presents with signs and symptoms of biliary obstruction due to obstruction of the biliary tree by adenocarcinoma in the head of the pancreas. Symptoms of pancreatic cancer in this patient include pain radiating to the back and weight loss. Diarrhea and steatorrhea result from fat malabsorption, which is secondary to extrahepatic obstruction to bile fl ow by encroachment of the tumor and metastatic lymph nodes on the common bile duct. Choices A, B, and D are incorrect because the ALT and AST were normal, which is unlikely in the setting of hepatitis. High serum alkaline phosphatase signals obstructive jaundice. Diagnosis: Extrahepatic biliary obstruction, pancreatic carcinoma

A 40-year-old woman presents with a 2-month history of burning epigastric pain that usually occurs between meals. The pain can be relieved with antacids or food. The patient also reports a recent history of tarry stools. She denies taking aspirin or NSAIDs. Laboratory studies show a microcytic, hypochromic anemia (serum hemoglobin = 8.5 g/dL). Gastroscopy reveals a bleeding mucosal defect in the antrum measuring 1.5 cm in diameter. An endoscopic biopsy shows that the lesion lacks mucosal lining cells and is composed of amorphous, cellular debris and numerous neutrophils. Which of the following is the most important factor in the pathogenesis of this patient's disease? (A) Achlorohydria (B) Acute ischemia (C) Autoimmunity (D) Gastrinoma (E) Helicobacter pylori infection

The answer is E: Helicobacter pylori infection. Peptic ulcer disease refers to breaks in the mucosa of the stomach and small intestine, principally the proximal duodenum, which are produced by the action of gastric secretions. The pathogenesis of peptic ulcer disease is believed to involve an underlying chronic gastritis caused by H. pylori. This pathogen has been isolated from the gastric antrum of virtually all patients with duodenal ulcers and from about 75% of those with gastric ulcers. H. pylori gastritis is the most common type of gastritis in the United States and is characterized by prominent chronic infl ammation of the antrum and body of the stomach. In addition to peptic ulcer disease, H. pylori gastritis is a risk factor for development of gastric adenocarcinoma and lymphoma. Eradication of H. pylori infection is curative of peptic ulcer disease in most patients. Gastrinoma (choice D) is a rare cause of peptic ulcers. Achlorhydria (choice A) is incorrect because the formation of peptic ulcers requires at least some gastric acid secretion. Diagnosis: Chronic infectious gastritis, peptic ulcer disease, gastric ulcer

A 58-year-old woman is brought to the emergency department 4 hours after vomiting blood and experiencing bloody stools. The patient was diagnosed with alcoholic cirrhosis 2 years ago. The patient subsequently goes into shock and expires. The histologic appearance of the esophagus at autopsy is shown in the image. Which of the following is the most likely underlying cause of hematemesis and hematochezia in this patient? (A) Alcoholic hepatitis (B) Ischemia of the gastric mucosa (C) Mallory-Weiss syndrome (D) Peptic ulcer disease (E) Portal hypertension

The answer is E: Portal hypertension. Esophageal varices are dilated (varicose) veins immediately beneath the mucosa, which are prone to rupture and hemorrhage. They arise in the lower third of the esophagus, most often in the setting of portal hypertension, secondary to cirrhosis. The lower esophageal veins are linked to the portal system through gastroesophageal anastomoses. If the portal blood pressure exceeds a critical level, these anastomoses become prominent. When varices become greater than 5 mm in diameter, they are likely to rupture, in which case life-threatening hemorrhage may ensue. The other choices are not associated with bleeding esophageal varices. Alcoholic hepatitis (choice A) by itself does not cause varices, but long-term alcohol abuse often leads to cirrhosis and esophageal varices. Diagnosis: Esophageal varices, cirrhosis

A 47-year-old woman presents with a 3-month history of vague upper abdominal pain after fatty meals, some abdominal distension, and frequent indigestion. Physical examination shows an obese woman (BMI = 30 kg/m2) with right upper quadrant tenderness. An ultrasound examination discloses multiple echogenic objects in the gallbladder. The opened gallbladder is shown in the image. Which of the following metabolic changes is most likely associated with the formation of gallstones in this patient? (A) Decreased hepatic bilirubin conjugation (B) Decreased serum albumin (C) Increased bilirubin uptake by the liver (D) Increased hepatic calcium secretion (E) Increased hepatic cholesterol secretion

The answer is E: Increased hepatic cholesterol secretion. Cholesterol stones are round or faceted, yellow to tan, and may be single or multiple. Risk factors for cholesterol stones include female sex, diabetes, pregnancy, and estrogen therapy. Solitary, yellow, hard gallstones are associated with bile that is supersaturated with cholesterol. During their reproductive years, women are up to three times more likely to develop cholesterol gallstones than men. If the bile contains excess cholesterol or is defi cient in bile acids, it becomes supersaturated with cholesterol and precipitates to form stones (lithogenic bile). In obese women, cholesterol secretion by the liver is increased. Impaired gallbladder motor function is another risk factor that leads to gallstone formation. In this case, stasis permits the formation of biliary sludge, which then progresses to macroscopic stones. Choices A and B are not associated with gallstones. Choices C and D are not physiologic. Diagnosis: Cholelithiasis, cholesterol gallstones

For the patient described in Question 1, which of the following pathophysiologic mechanisms is most directly associated with the development of ascites? (A) Decreased aldosterone secretion (B) Decreased intravascular volume (C) Hyperalbuminemia (D) Increased intravascular oncotic pressure (E) Increased portal hydrostatic pressure

The answer is E: Increased portal hydrostatic pressure. Ascites refers to the accumulation of fl uid in the peritoneal cavity, often caused by portal hypertension. In the setting of cirrhosis, decreased intravascular oncotic pressure due to hypoalbuminemia is also an important factor in the pathogenesis of ascites (see choice D). Aldosterone secretion (choice A) is increased in cirrhotic patients. Overall, imbalances in Starling forces lead to transudation of fl uid into the abdominal cavity. Diagnosis: Alcoholic cirrhosis, portal hypertension

A 60-year-old woman complains of increasing abdominal girth of 4 weeks in duration. Physical examination discloses ascites, and cytologic examination of the fl uid reveals malignant cells. Exploratory laparotomy shows multiple tumor nodules on the serosal surface of the intestines. Which of the following is the most likely diagnosis? (A) Carcinoid tumor (B) Gastrointestinal stromal tumor (C) Liposarcoma (D) MALToma (E) Metastatic carcinoma

The answer is E: Metastatic carcinoma. Metastatic carcinoma is by far the most common malignant disorder affecting the peritoneum. Ovarian, gastric, and pancreatic carcinomas are particularly likely to seed the peritoneum, but any intra-abdominal carcinoma can spread to the peritoneum. Metastatic carcinoma to the abdomen presents in the form of multiple serosal nodules and ascites fl uid that contains malignant cells. Diagnosis: Metastatic carcinoma

A 45-year-old woman complains of chronic, right lower quadrant pain. An abdominal CT scan reveals a globular, smooth-walled mass protruding into the cecum. The patient subsequently has the mass removed and the surgical specimen is shown in the image. Which of the following is the most likely diagnosis? (A) Acute appendicitis (B) Adenocarcinoma (C) Carcinoid tumor (D) Foreign body (E) Mucocele of the appendix

The answer is E: Mucocele of the appendix. Mucocele refers to a dilated mucous-fi lled appendix. The pathogenesis may be neoplastic or nonneoplastic. In the nonneoplastic variety, chronic obstruction leads to the retention of mucus in the appendiceal lumen. In the presence of a mucinous cystadenoma (in this case) or mucinous cystadenocarcinoma, the dilated appendix is lined by a villous adenomatous mucosa. A mucocele may become secondarily infected and rupture. Rupture of a neoplastic mucocele may seed the peritoneal cavity with mucus-secreting tumor cells, a condition referred to as "pseudomyxoma peritonei." Diagnosis: Mucocele

A 10-year-old boy is brought to the emergency room after 48 hours of fever and severe abdominal pain. He had developed edema of the legs several weeks previously. The temperature on admission is 38.7°C (103°F). Physical examination shows rebound tenderness, guarding, and ascites. An abdominal tap returns numerous segmented neutrophils. This child's spontaneous bacterial peritonitis is most often associated with which of the following underlying conditions? (A) Celiac sprue (B) Diverticulosis (C) Hirschsprung disease (D) Meconium ileus (E) Nephrotic syndrome

The answer is E: Nephrotic syndrome. Most cases of peritonitis are caused by bacteria that enter the abdominal cavity from a perforated viscus or through an abdominal wound. However, spontaneous bacterial peritonitis occurs in children without an obvious perforation. Most of these patients have a nephrotic syndrome and a systemic infection that seeds the ascitic fl uid with bacteria. In adults, spontaneous bacterial peritonitis is a feared complication of cirrhosis. The other choices are not associated with the development of spontaneous bacterial peritonitis. Diagnosis: Spontaneous bacterial peritonitis, nephrotic syndrome

A 45-year-old man describes burning epigastric pain 2 to 3 hours after eating. Foods, antacids, and over-the-counter medications provide no relief, and prescribed inhibitors of acid secretion are only moderately effective. Recently, the patient noticed that his stools were black. Physical examination reveals abdominal tenderness. The blood pressure is 120/80 mm Hg in the supine position and 90/50 mm Hg sitting up. The patient complains of lightheadedness upon returning to a standing position. CBC shows a hemoglobin of 6.3 g/dL. Endoscopy reveals multiple gastric and duodenal ulcers. Epigastric pain and anemia are most likely related to a neoplasm arising in which of the following anatomic locations? (A) Adrenal medulla (B) Ampulla of Vater (C) Duodenum (D) Esophagus (E) Pancreas

The answer is E: Pancreas. Zollinger-Ellison syndrome is characterized by unrelenting peptic ulceration in the stomach or duodenum (or even proximal jejunum) by the action of tumor-derived gastrin. Gastrin-producing neuroendocrine tumors (gastrinomas) usually arise in the pancreatic islets. Among islet cell tumors, pancreatic gastrinomas are second in frequency only to insulinomas. Duodenum (choice C) is incorrect because only 15% of cases of Zollinger-Ellison syndrome are due to gastrinomas outside the pancreas (e.g., duodenum). Most gastrinomas are malignant. Diagnosis: Zollinger-Ellison syndrome, peptic ulcer disease, gastric ulcer

A 38-year-old man is brought to the emergency room with clouded sensorium and lethargy. He had been degreasing the engine parts of an old car the previous day, using industrial solvents. Later that evening he felt ill, and by morning, he was diffi cult to rouse. Serum ALT is extremely high (2,400 U/L). He dies 2 days later in hepatic coma. Which of the following liver injuries is the most likely diagnosis? (A) Alcoholic hepatitis (B) Allergic reaction (C) Budd-Chiari syndrome (D) Idiosyncratic reaction (E) Predictable toxic liver injury

The answer is E: Predictable toxic liver injury. Acute, chemically induced hepatic injury spans the entire spectrum of liver disease, from transient cholestasis to massive hepatic necrosis. Drug-induced liver injury can be either direct or indirect. Indirect injury is caused by metabolites and free radicals that are produced as byproducts of xenobiotic metabolism. Immune reactions against a chemical or its metabolites are also causes of indirect liver damage. Chemically-induced hepatic injury is classifi ed as "predictable" when toxicity is immediate and dosedependent and as "unpredictable" or "idiosyncratic" when toxicity occurs without explanation (choice D). In this case, exposure to industrial solvents, such as carbon tetrachloride, caused predictable toxic liver injury, characterized by centrilobular necrosis and elevated serum levels of transaminases. Diagnosis: Toxic liver injury

A 42-year-old man is brought to the emergency room with right upper quadrant pain, shaking chills, and a fever of 38.7°C (103°F). His past medical history is signifi cant for an appendectomy 2 weeks previously. Physical examination reveals hepatomegaly and tenderness in the right upper quadrant. Laboratory studies show normal levels of serum albumin, ALT, and bilirubin, as well as increased alkaline phosphatase of 240 U/L. The WBC count is 28,000/μL. Which of the following is the most likely diagnosis? (A) Acute cholecystitis (B) Acute hepatitis (C) Diffuse peritonitis (D) Extrahepatic biliary obstruction (E) Pyogenic liver abscess

The answer is E: Pyogenic liver abscess. Pyogenic liver abscesses are caused by staphylococci, streptococci, and Gram-negative enterobacteria (i.e., anaerobic inhabitants of the gastrointestinal tract). The bacteria gain access to the liver by direct extension from contiguous organs or through the portal vein or hepatic artery. Extrahepatic biliary obstruction (choice D), which leads to ascending cholangitis, is the most common cause of pyogenic abscess and is usually associated with stones in the common bile duct (choledocholithiasis), benign and malignant tumors, or postsurgical strictures of the bile ducts. As in this instance of appendicitis, the infectious organisms can also originate within the abdomen and reach the liver by embolization. Other abdominal causes of pyogenic abscess in the liver include diverticulitis and infl ammatory bowel diseases. Diffuse peritonitis (choice C) is a possible complication of perforated appendicitis but is not suggested by the clinical presentation described in this vignette. Acute cholecystitis (choice A) is a very unlikely complication of appendicitis. Diagnosis: Pyogenic liver abscess

A 40-year-old black woman has frequent indigestion after meals and abdominal pain. Physical examination demonstrates a moderately obese woman in no acute distress. An ultrasound examination demonstrates numerous echogenic objects within the gallbladder. A cholecystectomy is performed, and the surgical specimen is shown in the image. The gallstones seen in this patient are typically associated with which of the following diseases? (A) Chronic pancreatitis (B) Diabetes mellitus (C) Familial hypercholesterolemia (D) Hyperparathyroidism (E) Sickle cell disease

The answer is E: Sickle cell disease. Black (pigmented) gallstones are irregular and measure less than 1 cm across. On cross section, the surface appears glassy. Black stones contain calcium bilirubinate, bilirubin polymers, calcium salts, and mucin. Hemolysis in patients with sickle cell disease or other chronic hemolytic anemias generates excess bilirubin, which predisposes to pigment stone formation. Cirrhosis, either through liver cell damage or hemolysis, predisposes to black stones. Gallstones can cause pancreatic duct obstruction, increasing the risk for development of acute and chronic pancreatitis (choice A). Hypercholesterolemia is a risk factor for development of cholesterol gallstones. Diagnosis: Cholelithiasis, sickle cell disease

An 85-year-old man complains of abdominal pain and bright red blood in his stool. An X-ray fi lm of the abdomen shows fecal impaction in the rectosigmoid region. Which of the following pathologic lesions is most likely to be found in this patient? (A) Curling ulcer (B) Cushing ulcer (C) Melanosis coli (D) Peptic ulcer (E) Stercoral ulcer

The answer is E: Stercoral ulcer. Incomplete evacuation of the feces, usually in association with debilitating disease or old age, may lead to the formation of a large mass of stool that cannot be passed, termed fecal impaction. Stercoral ulcers result from pressure necrosis of the mucosa caused by the fecal mass. Complications include rectal bleeding and perforation. The other ulcers do not occur in the rectum. Diagnosis: Stercoral ulcer

A 30-year-old woman presents with 2 days of abdominal cramping and diarrhea. Her temperature is 38°C (101°F), respirations are 32 per minute, and blood pressure is 100/65 mm Hg. Stool culture shows a toxigenic Escherichia coli infection. Which of the following best explains the pathogenicity of this organism in this patient? (A) Destruction of Peyer patches (B) Invasion of the mucosa of the colon (C) Invasion of the mucosa of the ileum (D) Stimulation of acute infl ammation in the superfi cial bowel mucosa (E) Stimulation of fl uid transport into the lumen of the intestine

The answer is E: Stimulation of fl uid transport into the lumen of the intestine. The most signifi cant factor in infectious diarrhea is increased intestinal secretion, stimulated by bacterial toxins and enteric hormones. Organisms that produce diarrhea by secreting specifi c toxins include Vibrio cholera and toxigenic strains of E. coli. There is minimal or absent damage to the intestinal mucosa (choices A to D) in cases of toxigenic diarrhea. The organisms remain attached to the intestinal mucosa and elaborate toxins, which stimulate the transmucosal transport of fl uid into the lumen, resulting in diarrhea. Patients may become severely dehydrated, particularly in the case of cholera. Diagnosis: Bacterial diarrhea

A 70-year-old man is rushed to the emergency room complaining of severe abdominal pain and rectal bleeding of 2 hours in duration. He has a history of coronary artery disease. Bowel sounds are absent on physical examination. A CT scan of the abdomen shows distention of the stomach and air-fl uid levels in the small bowel. Abdominal pain and bleeding in this patient most likely involved acute occlusion of which of the following arteries? (A) Celiac trunk (B) Gastroduodenal artery (C) Inferior mesenteric artery (D) Inferior rectal artery (E) Superior mesenteric artery

The answer is E: Superior mesenteric artery. Sudden occlusion of a large artery by thrombosis or embolization leads to small bowel infarction before collateral circulation comes into play. Depending on the size of the artery, infarction may be segmental or may lead to gangrene of virtually the entire small bowel. The small intestine, which is supplied by the superior mesenteric artery, is more likely to suffer transmural hemorrhagic infarction than the large intestine. The inferior mesenteric artery (choice C), which supplies blood to the colon, is a less common site for atherosclerotic embolization than the superior mesenteric artery because of the smaller size of the latter and its oblique origin from the aorta. Pathologically, ischemic bowel disease is classifi ed as occlusive or nonocclusive. Occlusive disease is caused by thrombi or emboli, whereas nonocclusive disease is secondary to arterial narrowing by atherosclerosis. The other choices are not specifi cally associated with small bowel ischemia. Diagnosis: Ischemic colitis

A 27-year-old woman presents with a 9-month history of bloody diarrhea and crampy abdominal pain. Three weeks ago, she noticed that her left knee was swollen, red, and painful. Her temperature is 38°C (101°F), respirations are 32 per minute, and blood pressure is 130/90 mm Hg. Abdominal palpation reveals tenderness over the left lower quadrant. Laboratory studies show moderate anemia, with a hemoglobin level of 9.3 g/dL. Microscopic examination of the stool reveals numerous red and white blood cells. A diffusely red, bleeding, friable colonic mucosa is visualized by colonoscopy. The colon is subsequently removed and the surgical specimen is shown in the image. Which of the following is the most likely diagnosis? (A) Adenocarcinoma (B) Carcinoid tumor (C) Crohn disease (D) Pseudomembranous colitis (E) Ulcerative colitis

The answer is E: Ulcerative colitis. Ulcerative colitis is an infl ammatory disease of the large intestine characterized by chronic diarrhea and rectal bleeding. It is associated with a pattern of remission and exacerbations and the possibility of serious local and systemic complications. The disorder occurs principally, but not exclusively, in young adults. Ulcerative colitis is essentially a disease of the mucosa. The process extends from the rectum for a variable distance proximally and is limited to the colon and rectum. Pseudomembranous colitis (choice D) is usually a complication of antibiotic therapy, and the mucosal surface of the colon is covered by raised, irregular plaques composed of necrotic debris and an acute infl ammatory exudate. Crohn disease (choice C) typically affects the colon in a patchy distribution with transmural infl ammation. Diagnosis: Ulcerative colitis

A 3-day-old neonate born after a 32-week gestation develops yellow skin. Physical examination of the infant is unremarkable. Which of the following is most likely to be increased in this neonate's serum? (A) Alanine aminotransferase (B) Carotene (C) Conjugated bilirubin (D) Galactosyltransferase (E) Unconjugated bilirubin

The answer is E: Unconjugated bilirubin. Approximately 70% of normal newborns exhibit transient unconjugated hyperbilirubinemia. Immaturity of the liver leads to inadequate conjugation of bilirubin. This physiologic jaundice is more pronounced in premature infants due to inadequate hepatic clearance of bilirubin and increased erythrocyte turnover. Fetal bilirubin levels in utero remain low because bilirubin crosses the placenta, where it is conjugated and excreted by the mother's liver. The other enzymes are unrelated to neonatal jaundice. Elevated serum levels of carotene (choice B) refl ects hypervitaminosis A. Diagnosis: Neonatal (physiologic) jaundice

A 63-year-old woman complains of rectal bleeding of 1 week in duration. Laboratory studies show hypochromic, microcytic anemia (hemoglobin = 7.6 g/dL and MCV = 70 μm3). Colonoscopy reveals a large polypoid mass, which is removed (surgical specimen shown in the image). The arrow points to a malignant tumor. The patient asks about the relative risk of cancer arising in various types of gastrointestinal polyps. Which of the following types of colonic polyps is most likely to undergo malignant transformation? (A) Hyperplastic polyp (B) Lymphoid polyp (C) Peutz-Jeghers polyp (D) Tubular adenoma (E) Villous adenoma

The answer is E: Villous adenoma. These polyps comprise one third of colonic adenomas and are found predominantly in the rectosigmoid region. They are typically large, broadbased, elevated lesions that display a shaggy, caulifl ower-like surface. Microscopically, villous adenomas are composed of thin, tall, fi ngerlike processes, which superfi cially resemble the villi of small intestine. Compared to tubular adenomas (choice D), villous adenomas more frequently contain foci of carcinoma. Hyperplastic polyps (choice A) have a much lower risk for malignant transformation. Diagnosis: Gastrointestinal polyp, villous adenoma

A 36-year-old man presents with fever and painful joints for 2 weeks. Physical examination shows skin pigmentation, glossitis, angular cheilitis, and generalized lymphadenopathy. The patient has lost 9 kg (20 lb) over the past 6 months. He reports that his stools are pale and foul smelling. Blood cultures are negative. The patient is started on antibiotic therapy and exhibits remarkable clinical improvement. Biopsy of the small intestine shows marked distortion of the intestinal villi, and a periodic acid-Schiff stain reveals large, foamy macrophages fi lled with glycoprotein-rich granules (shown in the image). Which of the following is the most likely diagnosis? (A) Angiodysplasia of ileum (B) Crohn disease (C) Ménétrier disease (D) Peutz-Jeghers syndrome (E) Whipple disease

The answer is E: Whipple disease. Whipple disease is a rare infectious disorder of the small intestine in which malabsorption is the most prominent feature. The disorder typically features infi ltration of the small bowel mucosa by macrophages that are packed with small, rod-shaped bacilli (Tropheryma whippelii). Infi ltrates of macrophages containing bacilli may be found in other organs, including regional lymph nodes and the heart. The other choices do not feature these distinctive aggregates of foamy macrophages. Diagnosis: Whipple disease

A 36-year-old woman presents with a 6-month history of progressive generalized itching, weight loss, fatigue, and yellow sclerae. She denies use of oral contraceptives or any other medication. Physical examination reveals mild jaundice and steatorrhea. Blood studies show a high cholesterol level of 350 mg/ dL, elevated serum alkaline phosphatase (240 U/L), and normal levels of AST and ALT. An intravenous cholangiogram shows no evidence of obstruction. An antimitochondrial antibody test is positive; antinuclear antibodies are not present. Which of the following skin manifestations is expected in this patient? (A) Acanthosis nigricans (B) Hyperpigmentation (C) Keratoacanthoma (D) Seborrheic keratosis (E) Xanthoma

The answer is E: Xanthoma. Primary biliary cirrhosis (PBC) is a chronic progressive liver disease that is associated with many immunologic abnormalities and is, therefore, widely held to be an autoimmune disease. The hallmark of this condition is the presence in serum of antimitochondrial antibodies. These autoantibodies recognize epitopes associated with the mitochondrial pyruvate dehydrogenase complex. Despite the specifi city of the antimitochondrial antibodies, they have no inhibitory effect on mitochondrial function and play no known role in the pathogenesis of the disease. The complement system is chronically activated. PBC occurs more often in women than in men (10:1 female predominance). It presents with fatigue, anorexia, jaundice, xanthomas of the skin, and pruritus. The other choices are not associated with PBC or hypercholesterolemia. Diagnosis: Primary biliary cirrhosis

A 9-year-old boy undergoes emergency surgery for presumptive acute appendicitis. During the operation, the surgeon notices that the boy's ileocecal lymph nodes are enlarged and matted together. One of the nodes is sent for a frozen section. The pathologist fi nds granulomatous infl ammation with central necrosis. The specimen is cultured. Which of the following pathogens is the most likely cause of lymphadenopathy in this patient? (A) Campylobacter jejuni (B) Shigella dysenteriae (C) Toxigenic E. coli (D) Vibrio cholerae (E) Yersinia enterocolitica

The answer is E: Yersinia enterocolitica. Yersinia can cause mesenteric adenitis and pain in the right lower quadrant (pseudoappendicitis). Infected children not infrequently have undergone laparotomy because of a mistaken diagnosis of appendicitis. The lymph nodes show a granulomatous infl ammation. Other symptoms are diarrhea, reactive arthritis, erythema nodosum, and septicemia. The other choices are not associated with the development of mesenteric adenitis and do not present with symptoms that mimic acute appendicitis. Diagnosis: Yersinia lymphadenitis

A 65-year-old woman complains of a 4-month history of bad breath, regurgitation of undigested food, occasional aspiration of food, and change in the sound of her voice. A barium swallow examination shows a posterior, midline pouch greater than 2 cm in diameter arising just above the cricopharyngeal muscle. Which of the following is the most likely diagnosis? (A) Epiphrenic diverticulum (B) Intramural pseudodiverticulum (C) Meckel diverticulum (D) Traction diverticulum (E) Zenker diverticulum

The answer is E: Zenker diverticulum. Zenker diverticulum is an uncommon lesion that appears high in the esophagus and affects men more than women. It was once believed to result from luminal pressure exerted in a structurally weak area and was, therefore, classed as a pulsion diverticulum. The cause is probably more complicated, but disordered function of the cricopharyngeal musculature is still thought to be involved in the pathogenesis of this false diverticulum. Most affected persons who come to medical attention are older than 60 years, an observation that supports the belief that Zenker diverticulum is an acquired lesion. Epiphrenic diverticuli (choice A) are located immediately above the diaphragm. Intramural pseudodiverticulum (choice B) is characterized by numerous small diverticula in the wall of the esophagus. Traction diverticuli (choice D) are outpouchings that occur principally in the midportion of the esophagus. Diagnosis: Zenker diverticulum

A 44-year-old woman complains of having yellow eyes, dark urine, and recurrent fever for about 3 months. She has a long history of chronic diarrhea. On physical examination, the patient is thin and jaundiced. The liver edge descends 1 cm below the right costal margin and is nontender. Laboratory studies show elevated serum bilirubin of 3.8 mg/dL, normal levels of AST and ALT, and an elevated level of alkaline phosphatase (440 U/dL). Endoscopic retrograde cholangiopancreatography demonstrates a beaded appearance of the extrahepatic biliary tree. Which of the following is the most likely underlying cause of diarrhea in this patient? (A) Amebiasis (B) Amyloidosis (C) Carcinoma of the ampulla of Vater (D) Celiac sprue (E) Ulcerative colitis

The correct answer is E: Ulcerative colitis. Primary sclerosing cholangitis (PSC) is characterized by infl ammation and obliterative fi brosis of intrahepatic and extrahepatic bile ducts. Approximately 70% of patients with PSC have longstanding ulcerative colitis, although the prevalence of PSC in such patients is only 4%. The clinicopathologic fi ndings are complemented by a characteristic radiographic appearance of a beaded biliary tree, representing sporadic strictures. The other choices are not associated with PSC. Diagnosis: Primary sclerosing cholangitis, ulcerative colitis


Ensembles d'études connexes

AWS sysops or architect 377 multiple choice questions by rajpalmanish

View Set

Assessment of Hematologic Function and Treatment Modalities (Chapter 32)

View Set

pharmacology week 7 quizes 86 87 88

View Set

Методи науково-педагогічних досліджень

View Set

vertebral column and thoracic cage TEST 3

View Set

Unit 13-16 (Practice Study Questions)

View Set